Download as pptx, pdf, or txt
Download as pptx, pdf, or txt
You are on page 1of 205

Our Lady of Fatima University

Valenzuela • Quezon City • Antipolo • Pampanga • Cabatuan

Mathematics in the
Modern World
Authors :
Clarenz Dalisay, LPT Grace Ann Delos Reyes, LPT,
Engr. Elise MATMath Cheryl Lumiguen,
Erandio Renato LPT, MAT
Nieva, LPT, MA Melanio Nipas Jr,
Rey-ann Rivera, LPT, LPT, MA Engr.
MSMath(c) Engr. Ernesto Angelica Sicat
Villarica

2
PREFACE

This is worktext is a collaborative efforts of the different faculty


members from Mathematics and Physics Department of all campuses
of this university. This will help the College students to appreciate
Mathematics as an important tool in their lives. This work will also
give them a new perception of Mathematics, not as just pure
calculation but instead a body of knowledge that can be seen
everywhere and add colors in their lives. This worktext is divided into
two parts that covers the mandated and elective topics that are
identified by the Commission of Higher Education (CHED) in the GE
Course.

3
Copyright, 2018
by:
Clarenz Dalisay, Grace Ann Delos Reyes, LPT,
LPT Engr. Elise MATMath Cheryl Lumiguen,
Erandio Renato LPT, MAT
Nieva, LPT, MA Melanio Nipas Jr,
Rey-ann Rivera, LPT, LPT, MA Engr.
MSMath(c) Engr. Ernesto Angelica Sicat
Villarica

Disclaimer: All literary works that appear on the book are copyrighted
by their respective owners. We claim no credit for them
unless otherwise noted. If you own the rights to any of the
works and do not wish them to appear on the book, please
contact us and they will be promptly removed.
4
TABLE OF CONTENTS

Title Page
Disclaimer
Preface
Table of Contents

UNIT Page
1 Nature of Mathematics: Mathematics in our 4
World
1.1 Patterns and Numbers in Nature and the 4
World
2. Fibonacci Sequence 8
3. Golden Ratio 12

2 Nature of Mathematics: Mathematical 15


Language and Symbols
2.1 Language of Mathematics 16
2. Sets 23
3. Elementary Logic 24

3 Nature of Mathematics: Problem 28


Solving and Reasoning

5
1. Inductive Reasoning 28
2. Deductive Reasoning 31
3. Polya’s Strategy 32

4 Mathematics as a Tool: Data Management 43


1. Statistics, Origin and Terms 43
2. Measure of Central Tendency 48
3. Measure of Variation 55
4. Quartile, Decile and Percentile 59
5. Normal Distribution 70
6. Correlation and Linear Regression 86

5 Mathematics as a Tool: Codes and 95


Cryptography
5.1 Coding Theory 95
5.2 Cryptography 115

6 Mathematics as a Tool: Apportionment and 127


Voting
6.1 Apportionment 127
6.2 Voting Methods 141

7 Mathematics as a Tool: Graph Theory 158


7.1 Basic Terms 158
7.2 Applications of Graph Theory 181

6
7
8
CHAPTER NATURE OF MATHEMATICS:

1 MATHEMATICS IN OUR WORLD

LEARNING OUTCOMES:

At the end of the chapter the students are expected to :

1. Argue about the nature of mathematics, what it is, how


it is expressed, presented and used.
2. Express appreciation for mathematics as human endeavor.

LESSON 1 : PATTERNS AND NUMBERS IN NATURE


AND THE WORLD
Humans developed a system for recognizing thoughts, classifying and
organizing patterns which are vital clues to the rules governing natural
processes. It is called Mathematics.

Early Greek Philosophers studied different patterns in attempt to explain the


order of nature. The humanity became explicitly aware of the two types of
patterns known as fractals and chaos. Fractals are geometric shape that
repeat their structure on an even finer scale, while chaos shows apparent
randomness whose origins are entirely deterministic. Pythagoras explained
the harmonies in music as arising from a number. Plato considered that there
is a universal pattern or ideal forms to which objects in nature are imperfect
copies. For example, a flower can be circular but not a mathematical circle.

World, by nature, is a combination of patterns and they can be seen by careful


observations in spite of regularities and irregularities. Any form of these
observations can be modelled mathematically. These natural patterns include
symmetry, spirals, tessellation, stripes, and so on.

9
Waves and dunes are clues to the flow of water, sand and air. Regular nightly
motion of stars also confirms that the Earth rotates, and that rotation was used
by ancients to predict time. Rainbows tells us the scattering of light, indirectly
confirming that raindrops are spheres. The tiger’s stripes and the hyena’s spots
attest to mathematical regularities in biological growth and form. Animals have
a mirror symmetry, so do leaves and some flowers. Echinoderms like starfish,
sea urchins and sea lilies have five-fold symmetry. Snowflakes have six-fold
symmetry, and a water splash approximates a radial symmetry. Considering
numbers as simplest mathematical objects, a year is roughly three hundred
sixty-five days, people have two legs, cats have four while spiders have eight.
Have you gone for a trip such as mountain climbing and noticed the world
around you? The differences in leaves, in hues of the sky, the transformations
and movements of clouds, how high a rabbit can jump nor how fast a turtle run?
Have you ever wondered how much water one must take? Are you watching
your weight and your food caloric intake? Or how much time you allot in
preparing things for such a trip? Have you not wondered how most vendors do
not make a mistake in giving your change after buying even without
calculators? Consciously or unconsciously, all of these activities engage some
form of mathematics. Mathematics aside from being related to numbers, is also
considering symbols, notations, operations and functions thus providing new
questions to think about. Pythagoreans obsession with number was not
baseless. Too many ‘coincidences’, too many connections—between number and
number, number and shape, number and music—led them to re-examine the
accepted world view of their day. Numbers, as originally conceived, were the
pragmatic progeny of accounting and commerce; but slowly, as they ‘grew’,
were seen by the Pythagoreans (and others) as having a meaning that stretched
far beyond the mere representation of ‘quantities’ and the collections of material
objects that had spawned their introduction. The Pythagorean vision marked a
fundamental turning point in the attitude not only to numbers, but indeed to
nature itself.

Nature has its laws. Natural laws are not like laws in our society. Societal laws
concerns human actions and are determined by governing bodies. Natural laws
on the other hand are determined by fundamental forces in the nature. These
arises from careful application of systematic studies of the natural world trough
experimentation and observation called as scientific methods. This method
provides scientist a rigorous framework to objectively

1
0
study the natural world. Natural laws can be verified through experiments
conducted by independent observers. The law can be a simple statement in
words, as well as a mathematical equation. Examples of these natural laws
are (1) Laws of Planetary Motion by Johannes Kepler, (2) Law of Universal
Expansion by Edwin Hubble, (3) Laws of Motion and Universal Gravitation by
Sir Isaac Newton, (4) Fluid Mechanics by Sir George Stokes, and even the
Theory of Relativity of Albert Einstein would not have gone farther without
mathematics.

Many breakthroughs in the field of sciences and engineering are already


evident. Theoretical investigations for treatment procedures by modelling and
simulation of biological processes were made possible through creative
employment of mathematics. Improvements in Information and
Communication Technology also contributed to the fast pacing on human
endeavours in terms of communications, commerce and lifestyles. However,
though “millennials” are hesitant in considering courses that involve
mathematics, they are unknowingly applying mathematics as they show their
interests into “being connected” as they use gadgets and other technologies
appealing to their senses.

One can never deny the fact that mathematics is everywhere not only because
it can be seen in nature but also because of its practical applications in our
daily lives.

11
12
FIBONACCI SEQUENCE

Fibonacci sequence, named after Leonardo Pisano or Leonardo of Pisa, is a series


of numbers where a number is found by adding the two numbers before it.
Starting with 0 and 1, the succession goes on 0,1,1,2,3,5,8,13,21,34 and so on.

As a trivia, Fibonacci is a shortened word for the Latin term “filiusBonnaci” which
means “son of Bonaccio”. His father’s name was Guglielmo Bonnacio who was a
wealthy Italian merchant, representing the Republic of Pisa trading in Bugia,
Algeria in North Africa. Fibonacci was an extensive traveller around
Mediterranean and met numerous merchants. He studied different numerical
systems and methods, including the Hindu- Arabic numeral system which he
learned from North Africa and eventually making it as popular to Europe through
his Liber Abaci (Book of Calculation). Today, the system is the most common
representation of numbers.

The original problem Fibonnacci investigated was about how fast rabbits can
breed in ideal circumstances. Beginning with a newborn male and female rabbit,
we will assume the following conditions:

a. Rabbits reach sexual maturity after one month.


b. The gestation period for a rabbit is one month.
c. Female rabbit always gives birth to one male and one female rabbit every
month.
d. Rabbits do not die.

This problem can be illustrated as:

13
Considering the conditions given, the newborn pair of rabbit are not yet at
sexual maturity after one month, therefore, they can’t mate. At two months,
they have mated but not yet given birth resulting to having the same number
of pairs. After three months, the female rabbit will give birth to a pair,
resulting in two pairs. At the fourth month, the original pair gives birth again,
the second pair mates but not give birth, resulting to three pairs in all.
Continuing this process, this scenario answered Fibonacci’s question as to how
many pair of rabbits could be born in a year.

Fibonacci in Computer Science

The Fibonacci numbers also occur as the sums of “shallow” diagonals in


Pascal’s Triangle. Fibonacci numbers is also a complete sequence where every
positive integer can be written as a sum of Fibonacci numbers, where one
number is used once at most. In computer science, a search technique was
developed for searching a sorted array with aid from the sequence.

Fibonacci and several Biological Settings

Fibonacci sequence can be found in other places in nature like branching in


trees, arrangement of leaves on a stem, the fruitlets of a pineapple, the
flowering of artichoke, an uncurling fern and the arrangement of a pine cone.
Also on many plants, the number of petals is a Fibonacci number. Many
plants including buttercups have 5 petals; lilies and iris have 3 petals; some
delphiniums have 8; corn marigolds have 13 petals; some asters have 21
whereas daisies can be found with 34, 55 or even 89 petals.

14
The Fibonacci numbers are also well represented in honeybees. Honeybee
lives in a colony called hive and they have an unusual Family tree. In a
colony, there is one special female called queen. There are male bees who
does not work and are produced by the queen’s unfertilized eggs called
drones. They only have a mother but no father. There are also other female
bees who are called worker but they do not produce eggs. Female bees are
produced when the queen mated with a male bee so they have two parents.
They usually end up being a worker bees but some are being fed with a
substance called royal jelly which makes them grow into queens ready to go
off when the bees are ready to swarm and leave their home in search for a
new place to build their hive. If you take any bee hive and follow the pattern,
it would look like:
Number of Parents Grand Great Gt-Gt Gt-Gt-Gt
Parents Gran Gran Grand
d d Parent
Parent Parent s
s s
Male Bees 1 2 3 5 8
Female 2 3 5 8 13
Bees
Since a drone has only one parent (only a mother and no father), it also has
an interesting genealogical tree. Generation 1 has 1 member (the male). One
generation back there is also 1 member (the mother). Two generations back
there are 2 members (the mother and father of the mother). The number of
members in each generation follow the Fibonacci sequence.

15
Fibonacci Rectangles

Start with two small squares of size 1 next to each other,


draw on top of both of these a square of size 2 (=1+1).
Touching both a unit square and the latest square of
side 2, having sides 3 units long; and then another
touching both the 2-square and the 3-square (which has
sides of 5 units). Adding squares whose side is as long
as the sum of the latest two square's sides produces a
new rectangular image that follows Fibonacci sequence.
This set of rectangles whose sides are two
successive Fibonacci numbers in length and which are composed of squares with
sides which are Fibonacci numbers, we will call the Fibonacci Rectangles.

In each square of Fibonacci Rectangles, a spiral can be


drawn by putting together the quarters of a circle in
each square. The spiral is not a true mathematical spiral
(since it is made up of fragments which are parts of
circles and does not go on getting smaller and smaller)
but is a good approximation to a kind of spiral that does
appear often in nature. Such spirals are seen in the
shape of shells of snails and sea shells and in the
arrangement of seeds on flowering plants too. The
distance of the line from the center of the spiral
increase by a factor of the golden number in each
square. So points on the spiral are
1.618 times as far from the center after a quarter-turn, that is, equivalent to
6.854 times further out than when the curve last crossed the same radial line.

16
FIBONACCI SEQUENCE AND THE GOLDEN RATIO

Two consecutive Fibonacci numbers approaches Golden Ratio. It is easier to


see what is happening if we plot the ratios on a graph where Fib(i) can be
any Fibonacci number and Fib(i-1) is any number preceding it:

The golden ratio often represented by a Greek letter phi with a value of
1.618034 is also called the golden section or the golden mean or Divine
Proportion. The proportion is also said to be aesthetically pleasing due to
which several artists and architects, including Salvador Dali and Le Corbusier,
have proportioned their work close to the golden ratio. The Fibonacci
sequence and the golden ratio are intimately interconnected.

17
18
19
CHAPTER NATURE OF MATHEMATICS:

2 MATHEMATICAL LANGUAGE
ANd SYMBOLS

LEARNING OUTCOMES:

At the end of the chapter the students are expected to :

1. Discuss the language, symbols and conventions of


mathematics.
2. Explain the nature of Mathematics as a language.
3. Discuss the language, symbols and conventions of
mathematics.
4. Perform operations on Mathematical expressions
completely.
5. Acknowledge that mathematics is a useful language.

20
LESSON 1 : LANGUAGE OF MATHEMATICS
Language is a fundamental tool that bridges the gap among people from
varying origins and cultures without prejudice to their background and
upbringing. It can be communicated in either spoken or written manner as
long as words are arranged in a structured and systematic way.

As a general concept, “language” may refer to the ability to learn and use
systems of complex communications or to describe the set of rules that
makes up these systems.

Considering the Mandarin Language, a different sets of characters are


available for the sun, moon, stars, house, chair, table, trees, plants, flowers
and relationships like father, mother, brother and sister. These unfamiliar
characters may make learning Mandarin more difficult that the Greek
Language. Even the Greek letters are different from the English alphabet.
Mathematics is also a language. It has its own symbol system, the same way
as English or Greek languages have their own alphabet.

For example, try solving the given problem:

Ana est deux fois plus âgée que son frère et la somme de leurs âges est de 36
ans. Quel âge ont-ils?

Since the language used may seem unfamiliar to you, understanding the
entire problem creates another problem: failure to understand what the given
is all about. To understand the ideas or concepts of Mathematics and figure
out clearly the logic behind problems is one of the many reasons why we
need to know the language of mathematics. The language itself is precise, it
can make very fine distinctions among a set of symbols. It is concise because
it can briefly express long sentences. It is also powerful because of the
relative ease it gives upon expressing complex thoughts.

21
Mathematics is a symbolic language. Some of the symbols you may encounter
as you read this book are the following:

∑ The sum of
Ǝ There exists
Ɐ For every (for any)
∈ Element of (or member of)
∉ Not an element of (or not a member
of)
⊆ Subset of
⇒ If … , then
⟺ If and only if
ℜ Set of real numbers
N Set of natural number
Z Set of integers
Q Set of rational numbers
∞ Infinity

The subsets of the real world can be described using this mathematical
language symbols above. Problems in Physics, like the free-falling bodies,
speed and acceleration, quantities like the chemical content of vegetables,
the use of mathematical modelling in biological disease modelling and the
formula employed in the social sciences can all be expresses using
mathematical sentences. Even abstract structures can also be described by
mathematics. Examples of such are abstract algebra, linear algebra, topology,
real analysis and complex analysis, all of which have no known physical
counterparts.

Mathematics, therefore, is the language of the sciences, business, economics,


music, architecture, the arts, and even politics. There is an intimate
connection between the language of mathematics and the English language.
The brain hemisphere responsible for controlling language is also the same
hemisphere responsible for the tasks involving mathematics. This part of the
brain is the left

22
brain hemisphere that coordinates logical or analytical thinking while the right
brain hemisphere is responsible for creative thinking.

English Language VS Mathematics Language

ENGLISH MATHEMATICS
SYMBOLS English Alphabet English Alphabet, Numerals,
and punctuation Greek
Letters, Grouping Symbols,
Special Symbols
Name Noun Expressions
Complete Sentence Sentence
thought
Action Verbs Operations and other actions
(e.g. simplify, rationalize)
What is in a Verbs Equality, inequality,
sentence membership in a
set
Attribute of a Fact or fiction True or false
sentence
Synonyms Different words The same object but different
but names
the same
meaning

The table below shows the similarities or comparison of English language


and Mathematics.

2
3
Comparison: English Language VS Mathematics
Language

I. Name
In English, a word that functions as the name of a thing or a set of
things that serve as the main word in the subject is called Noun.
For simplicity, we will simply use it in this bookas the name of
things we want to talk about.

Example: a. Carol loves Mathematics.

However, in Mathematics, instead of using Noun, we call it


Expression which simply refers to the object of interest. Other
types of expressions are Numbers, set, functions, ordered pairs,
matrices, vectors, groups.

Example: a. 5

b. 1.2 + 6,

c.3x – 3

d.ordered pair
(a, b)

e.a function f(x)

II. Complete Thought


Both in English and Mathematics that a Sentence must show
complete
thought (noun and verb) and express true, false or sometimes true
or sometimes false idea. Mathematical sentences however may
neither be true or false but not both.

Example: a. He loves Mathematics.

b. 1 + 3
=4

24
III. Synonyms
In English, Synonyms are different words with the same meaning.

Example: Group - association

In Mathematics, Synonyms are the same object but with different


names.

Example: 1 + 2 + 5 and 8 ½ + ½, 2 - 1, 5/5

Some Difficulties in the Math Language

1.Different meaning/use of words in Math and English

“and” is equivalent to plus

“is” may have different

meaning

2.The different uses of numbers: cardinal, ordinal or nominal

Cardinal numbers are the ones used for counting, ordinal numbers are
those that tells the position of a thing while nominal numbers are
numbers used only as a name to identify something, not as an actual
value or position.

3.Mathematical objects may be expressed in many ways such as sets


and functions.

25
Translation: English Language to Math Language and vice versa

I. English to Math
English Mathematics

1. The sum of a number and 10 X + 10


2. The product of two numbers x •y
3. The product of -1 and a -1 • x
number
4. One-half times the sum of ½ (X + Y)
two
numbers
5. Twice a number 2X

Practice:

1. 1. Five less than a number


2. A number, less 8
3. Six more than a number
4. A number, plus 6
5. The square of a number
6. Four times the square of a number
7. One –half a number
8. Three less than twice a number
9. Five more than three a number
10.The square of the sum of 5 and a
number

26
II. Mathematics to English
Choose a quantity to be represented by a variable, then write the
mathematical expression for each number.

1. A three-digit numbers whose hundreds digit is half the tens digit and
the tens digit is 2 more than the units digit.
Let x = be the unit’s value

½ (x + 2) x+2 x

2. The total interest earned after one year when P 100 000 is invested
part at 6 % annual interest rate and the remaining part at 7.5 %
annual interest rate.

Let x = be the part to be invested to at


7.5% y = be the total earned interest

(P 100 000 – x)0.06 + 0.075x = y

27
LESSON 2 : SETS

A set is a collection of numbers, peoples, letters of the alphabet, any other


sets and other distinct objects. They are conventionally represented by capital
letters. These objects are called elements. Sets can be described or specified
either by using a description or by listing each member of the set. That is,

A is the set of the first four positive integers

A = {1, 2, 3, 4}

Note that in listing, the order of the elements is irrelevant and in case that
the set has many elements, the listing can be abbreviated.

B is the set of the first thousand positive

integers B = {1, 2, 3, ... , 1000}

Considering the given sets A and B, every member of set A is also a member
of set B, therefore, it can also be stated as “A is a subset of B” or “A is
contained in B”. The relation between sets A and B is established by ⊆ is
called inclusion or containment.

If, however, there is another number of sets available and they are all
contained in B, set B will be considered as Universal set.

Also, a set with no element is called null set.

28
Lesson 3 : Elementary logic

Logic allows us to determine the validity of arguments in and out


of mathematics.

Proposition is a declarative statements that may expressed an idea which can


be true or false but not both. They can be expressed by the symbols P, Q, R
or p, q, r.

Types of Propositions

1.Simple – means single idea statement

2.Compounds – conveys two or more ideas.

Example:

Each of the following statement is a proposition. Some are true and some
are false. Can you tell which are true and which are false? If it is false,
state why.

a.9 is a prime number


b. 5 + 3 = 8
c. X2 + y2≥ 0
d. 10 < -3

Answers:

a. False, Prime numbers have no other factors than 1 and itself


b. True
c. True
d. False, A negative number is always less than a positive
number

29
Logical Connectives

Statements Connective Symbolic Form Type of


Statements

Not P Not ~P Negation

P and Q And P ∧Q Conjunction

P or Q Or P ∨Q Disjunction

If P, then q If…. Then P→ Q Conditional

P if and only if If and only if P ↔Q Bi-conditional


q

Quantifier

1. Universal Quantifier “ for all” or “for every” , denoted by


2. Existential Quantifier “ there exists” denoted by ∃

Converse, Inverse and Contrapositive of a Conditional

The Converse of p→q is q →q


The Inverse of p→q is ~p →~q
The Contrapositive p →q is ~q →~p
of

30
Examples of Logic

A. Write the following in symbolic form using P, Q, R statements and


the syml ~,→,↔,∨,∧ where

P: The sun is shining

Q: It is raining

R: The ground is wet

1.If it is raining then the sun is not shining


Answer: Q →~P

2.It is raining and the ground is wet.


Answer: Q ∧ R

3. The ground is wet if and only if it is raining and the


sun is shining.
Answer: R ↔ (Q ∧ P)

B. Write the statementusingⱯand ∃ as needed.

1. Everyone in the room is a registered student


.
2. Not all men are mortal.

Answer:

1. ⱯPerson x in the room, x is registered


nurse.

2. ∃ men x such that x is immortal.

31
32
CHAPTER
NATURE OF MATHEMATICS:

3 PROBLEM SOLVING AND


REASONING

LEARNING
OUTCOMES:

At the end of the chapter the students are expected to :

1. Apply inductive and deductive reasoning to solve problems.


2. Solve problems involving patterns and recreational problems following
Polya’s
Strategy.
3. Organize one’s method and approaches for proving and solving
problems.

SOLVING PROBLEMS BY INDUCTIVE REASONING


Egyptian’s and Babylonian’s approach in solving problems was an example of
“do thus and so” method, that is, to solve a problem or perform an operation,
a cookbook-like recipe was given, and it was performed over and over to
solve similar problems.

They concluded the same method would work for any similar type of problem
by observing that a specific method worked for a certain type of problem.
Such conclusion is called conjecture.

A conjecture is an educated guess based on repeated observations of a


particular process or pattern. The method of reasoning we have described is
called inductive reasoning, where we use specific examples to reach a general
conclusion of something. However, it should be carefully noted that a
conjecture is an idea that may or may not be correct.

33
Use inductive reasoning to predict the next number.

1. Consider the counting numbers: 1,2,3,4, 5,…

Solution:

The three dots indicate that the numbers continue indefinitely in the
pattern established. By observing, we can see that the pattern adds 1
to the previous number to get the next number. So by applying
inductive reasoning, we can conclude that the next number would be
6.

2. 1, 3, 6, 10, 15, ?

Solution:

The first two numbers have a difference of 2. The second and third has
a difference of 3. The third and fourth, has
4. From this we can see that the
difference between any two numbers is
1 more than the preceding difference.
Since 10 and 15 differs by 5, we can
predict that the next number will be 6
larger than 15, that is, 21.

3. Now, let’s consider the following list of


natural numbers,
2, 9, 16, 23, 30,…

By applying the same reasoning we


used on the first example, we might
conclude
that the next number would be 37 by observing that any number in the
list is 7 more than the preceding number. As we have said, the
conjecture may or may not be correct.

34
For this example, you were tricked into coming up with a wrong
conclusion. Not that your logic was faulty, but the person who makes
the list thinks of another scenario. The numbers given are actually
dates of Mondays of June where the 1st day falls on a Sunday. From
this, the answer should have been 7, since the next Monday that
follows the 30th of June is 7th of July.

The process we used in predicting the next number for examples 1 and 2 may
reveal a flaw for our third example. We can never be sure that what is true to
a specific case will be true in general. Inductive reasoning does not guarantee
a true result, but it provide a means of arriving into a conclusion. A statement
that disproves the conjecture is called counterexample.

4. Verify that the statement x2> x if false by finding a


counterexample. Solution:
For x =1, we have 12 = 1. Since 1 is not greater than 1, we can say
that this
is a counterexample for the given problem. Thus, “for all numbers x,
x2> x”
is a false statement.

Practice:

Verify that each of the following statements is a false statement by finding a


counterexample for each.

𝑥
a. =
𝑥
1 𝑥+3
b =𝑥+
3
. 1
c. √𝑥2 + 16 = 𝑥 +
4

35
SOLVING PROBLEMS BY DEDUCTIVE REASONING

Deductive reasoning is the process of solving problems by applying premises,


syllogisms, and conclusions. It goes from general case to a specific case.

Key Terms:

Argument - is the reason or reasons offered for or against something


Premises - minor or major propositions or assertions that serve as the bases
for an argument. It can be an assumption, law, rule, widely held idea or
observation.
Syllogism - an argument composed of two statements or premises followed by
a conclusion.
Conclusion - the last step in a reasoning process

1. “Today is Thursday. Tomorrow is Friday.”

Solution:
There is only one premise in this statement, “Today is Thursday”. The
other statement “Tomorrow is Friday” is called conclusion. Following
the days in
a week, the fact that Friday is the day after Thursday is not explicitly
stated. Since the conclusion comes from general facts that apply to
this case, deductive reasoning was used.

2. All men are


mortal. Socrates
is a man.
Therefore, Socrates is mortal.

Solution: This example is reasoning that uses syllogism. The first two
statements are the premises and the third one is the conclusion.

3. All doctors are men.


My mother is a
doctor
Therefore, my mother is a man.

36
Solution: Saying that an argument is valid does not mean the
conclusion is true. Thus, if the premises are wrong, the argument
may be valid but the conclusion may not be true.

4. General case : “In any right triangle, the sum of the squares of the legs is
equal to the square of the hypotenuse”- Pythagorean Theorem
Specific case: Given: a = 3, b = 4
Problem: Find the value of the hypotenuse, c

Solution: Applying the Pythagorean Theorem to find the value of the


hypotenuse, we will use the mathematical translation of the theorem,
that is, a2 + b2 = c2. Working out this formula to get c, we have, 𝑐 =
√𝑎2 + 𝑏2.
Substituting the values of a and b as indicated from the given, we can
conclude that c = 5.

SOLVING PROBLEMS BY POLYA’S STRATEGY

POLYA’S STRATEGY named after George Polya (1887 – 1985), is a four-step


problem solving strategy that are deceptively simple.

Polya's four-step approach to problem solving

1. Preparation: Understand the problem

∙ Learn the necessary underlying mathematical concepts


∙ Consider the terminology and notation used in the problem:

1. What sort of a problem is it?


2. What is being asked?
3. What do the terms mean?
4. Are the information given enough?
5. What is known or unknown? Rephrase the problem in words.
6. Note specific examples of the conditions given in the problem.

37
2. Thinking Time: Devise a plan

∙ You must start somewhere so try something. How are you going to
attack the problem?
∙ Possible strategies: (i. e. reach into your bag of tricks.)
1. Make a list of the known and unknown information.
2. Use a variable for unknowns.
3. Draw pictures, diagrams or even tables.
4. Be systematic.
5. Solve a simpler version of the problem.
6. Guess and check. Trial and error. Guess and test. (Guessing is OK.)
7. Look for a pattern or patterns.
8. Work backwards. Guess at a solution and check your result.

Tip! ☺
✔ Once you understand what the problem is, if you are stumped or stuck,
set
the problem aside for a while. Your subconscious mind may keep
working on it.

3. Insight: Carry out the plan

Work out an idea or a new approach carefully and see if it leads to a solution.

If the plan does not seem to be working, then start over and try another
approach. Often the first approach does not work. Do not worry, just because
an approach does not work, it does not mean you did it wrong. You actually
accomplished something, knowing a way does not work is part of the process
of elimination. Remember to keep an accurate and neat record of all your
attempts.

The key is to keep trying until something works.

38
4. Verification: Look back and review your solution

∙ Check to see if your potential solution it works.

1.Did you answer the question?


2.Is your result reasonable?
3.Double check to make sure that all of the conditions related to the
problem are satisfied.
4.Review your computations in finding the solution.

∙ If your solution does not work, there may only be a simple mistake.
Try to modify your current attempt before scrapping it. Though you
have to remember likely that at least part of it will end up being
useful.
∙ Is there a simpler way to solve the problem? (You need to become
flexible in your thinking. There could be another way.)
∙ Can the problem or method be generalized so as to be useful for future
problems?

“A great discovery solves a great problem but there is a grain of discovery in


the solution of any problem. Your problem may be modest; but if it
challenges your curiosity and brings into play your inventive faculties, and if
you solve it by your own means, you may experience the tension and enjoy
the triumph of discovery.”
– George Polya

Examples:

1. The sum of two numbers is 30. The first number is twice as large as
the second number. What are the numbers?

39
Solution:

Understand the Problem: We need to determine two distinct numbers which


when added will give a sum of 30, and when compared, one is twice the
other.

Devise a Plan: Let x be the value of the second number and since the first
is twice the second, we will represent it as 2x.

Carry out the Plan:

2𝑥 + 𝑥 = 30
3𝑥 = 30
So, the second number is 𝑥 = 10 and the first number is 2𝑥 = 20.

Review the Solution:

Since the problem requires two numbers whose sum is 30, adding the two
numbers 20 and 10 gives 30. Also, 20 is twice of 10. Therefore, our
answer satisfies the conditions given in our problem.

2. In tossing two coins at a time, what is the probability of having 2 heads


in a single throw?

Solution:

Understand the Problem: A coin has two faces, a head and a tail
respectively. Two coins when tossed can show several combination. As
per required, the two coins should be thrown at once and they should
both show heads up.

Devise a Plan: Let H represent heads and T for tails. We are going to list
down all possible combinations without duplication.

40
Carry out the Plan: We will start to list the possibilities having the First coin
show heads and then tail. That is,

First Coin Second Coin The probability that will show both
H H heads will be computed as the
H T number of possibilities of having
two heads divided by the number
T H
of
T T trials, that is 1 𝑥 100% = 25%.
4

Review the Solution:

The list is organized and has no duplication, so out of a single toss of two
coins, there is only a 25% probability or chance of having two heads.

41
42
Score:
Date:

43
44
45
46
47
CHAPTER MATHEMATICS AS A TOOL :
DATA MANAGEMENT
4
LEARNING OUTCOMES:

At the end of the chapter the students are expected to :

∙ Use a variety of statistical tools to process and manage


numerical data.
∙ Compute the z-scores of a given data.
∙ Locate and calculate the different values of
measures of relative position and interpret these
values.
∙ Rule out relationship between two different variables and
∙ Use the correlation coefficient in making predictions
∙ Show knowledge on normal distribution.
∙ Interpret in standard deviation units any particular
score in normal distribution

LESSON 1 : STATISTICS, ORIGIN AND TERMS


Statistics itself came from the Latin word “status” which means state. From
the ancient times, statistics was used by state leaders to know how much tax
to levy their subjects and how many soldiers are needed in an expected war.

In capitalism, no also the leaders of the state but also capitalists, are
interested in statistical surveys resulting to increased demand for data
processing for their increasing benefits such as insurance.

48
STATISTICAL DATA

Data in statistics is always a result of experiment, observation, investigation


and other means and often appears as a numerical figure and then evaluated
to make it into useful knowledge.

For most people, “statistics” is a scary thing that must be avoided as much as
possible because they think that it is a collection of numbers and vague
formulas. People, without noticing, applies statistics in their everyday life,
such as to the amount of food they eat, how much money was in their purse,
how far is their work area to respective homes, color of their hair, number of
rebounds and assists of a player in a basketball game, their height, gender,
and so on. Statisticians develop and apply appropriate methods in collecting
and analyzing data. They guide the design of the research study and then
analyze the results. The interpretation of the result is the basis of the
statistician in making inferences about the population.
BASIC CONCEPTS

1. Descriptive Statistics – deals with the collection and presentation of


data and collection of summarizing values to describe its group
characteristics. The most common summarizing values are the
measure of central tendency and variation.
2. Inferential Statistics – deals with the predictions and inferences based
on the analysis and interpretation of the results of the information
gathered by the
statistician. Some of the common statistical tools of inferential
statistics are the t-test, z-test, analysis of variance, chi-square, and
Pearson r.
3. Variable – a numerical characteristic or attribute associated with the
population being studied. They are further classified as categorical or
qualitative and numerical or quantitative.
4. Discrete variables – values obtained by counting.
5. Continuous variables – values obtained by measuring, all of which
cannot be put into a list because they can have any value in some
interval of real numbers.

49
6. Scales of Measurement – subdivided into four categories and upon
drawing inferences on a random sample, the type of measurement
scale must be carefully chosen.
a. Nominal – classifies elements into two or more categories or
classes, the numbers indicating that the elements are different but
not according to order or magnitude
b. Ordinal –a scale that ranks individual in terms of the degree to
which they possess a characteristic of interest
c. Interval – in addition to ordering scores from high to low, it also
establishes a uniform unit in the scale so that any equal distance
between two scores is of
equal magnitude. There is no absolute zero in this scale.
d. Ratio – in addition to being an interval scale, it also has an absolute
zero.
7. Population – defined as groups of people, animals, places, things or
ideas to which any conclusions based on characteristics of a sample
will be applied.
8. Sample – a subgroup of the population.
9. Parameter – a numerical measure that describes a characteristic of the
population.
10. Statistic – numerical measure that is used to describe a characteristic
of a sample.

50
51
52
LESSON 2: MEASURES OF CENTRAL TENDENCY
To describe a whole set of data with a single value that represents the middle
or centre of its distribution is the purpose of measure of central tendency
(measures of centre or central location). To put in other words, it is a way to
describe the center of a data set.

Why Is Central Tendency Important?

∙ It lets us know what is normal or 'average' for a set of data.


∙ It also condenses the data set down to one representative value, which
is useful when you are working with large amounts of data.
∙ Central tendency also allows the comparison of one data set to
another, as well as one piece of data to the entire data set. For
example, you could easily draw comparisons between the girls’ and
boys’ heights by calculating the average height for each sample group.

Three main measures of central tendency

∙ Mean
∙ Median
∙ Mode

1. Mean (balance point)


∙ The mean by definition is the sum of all the values in the observation
or a dataset divided by the total number of observations. This is also
known as the arithmetic average.
∙ The mean can be used for both continuous and discrete numeric data
as well as for categorical data, as the values cannot be summed.
∙ As the mean includes every value in the distribution the mean is
influenced by outliers (which are numbers that are much higher or
much lower than the rest of the data set) and skewed (asymmetric)
distributions.
∙ This measurement is applicable to use for ratio and interval data.

53
Examples:

1. Find the mean of the following numbers: 47, 53, 67

Solution:
From the given problem, we can see that there are only 3 given
values. This is considered as ungroup. Therefore,

Mean = (47+53+67)/3 = 55.67

2. Find the mean of the following math test

scores:

33,44,55,65,55,33,67,78,54,55,55,44

Solution:
There are 12 observations or values in this given problem. Usually,
data are grouped if they are above 30 observations. We can work
this case either by using the grouped or ungrouped formula:
Using Ungrouped data

sum of observations 638


Mean = = x̄ = 53.17
total number of observations
12

54
Using Grouped Data

Test scores Frequency


33 2
44 2
55 4
65 1
67 1
78 1
54 1
N = 12

∑𝑓𝑥
x̄ = 𝑁

(33∗2)+(44∗2)+(55∗4)+(65∗1)+(67∗1)+(78∗1)+(5
x= 4∗1) 12

x̄= 53.17

2. Median (physical middle point)

∙ The median is considered as the physical middle point in a distribution


because it is located at the center position when the values are
arranged in ascending or descending order, which in turn divides the
distribution in half (there are 50% of observations on either side of the
median value).
∙ If a distribution has an odd number of observations, the median value
is the middle value.
If it is an even number, the median value is the mean or average of
the two middle
values.
∙ The median is usually the preferred measure of central tendency when
the distribution is not symmetrical because it is less affected by
outliers and skewed data than the mean.
∙ The median cannot be identified for categorical nominal data, as it
cannot be logically ordered.
∙ This is widely used for ordinal type of information.

55
Examples:

1. (a) Ungrouped data: 3,1,5,2,2,7,5,6,8

Arrange to ascending order: 1,2,2,3,5,5,6,7,8

There are 9 values in the distribution, therefore the median is the


one on the fifth position based on the arranged values: Median is 5

(b) Ungrouped data: 3,1,5,2,2,7,5,6,8,4

Arrange to ascending order: 1,2,2,3,4,5,5,6,7,8

Since there are 10 values in the given, we will take the average of
the fifth and sixth numbers in the arrangement, which is 4 and 5
respectively, therefore, Median is 4.5.

56
2. Find the median of the following math test scores.

Test scores Frequency


25-30 4
31-36 7
37-42 3
43-48 5
49-54 8
55-60 12
61-66 6
N = 45

Solution:

n/2 = 45/2 = 22.5

Test scores frequency <cf


25-30 4 4
31-36 7 11
37-42 3 14
43-48 5 19
49- 8 27
54
55-60 12 39
61-66 6 45
N = 45

57
3. Mode
∙ The mode can be found for both numerical and categorical (non-
numerical) data. It is the most commonly occurring value in a
distribution. There can be more than one mode for the same
distribution of data, (bi-modal, or multi-modal), thus limiting the
ability of the mode in describing the center of the distribution.
∙ In some particular cases, the distribution may have no mode at all
(i.e. if all values are different).In such case, it may be better to
consider using the median or mean, or group the data in to
appropriate intervals, and find the modal class.

Examples:

1. Ungrouped Data

a. 33,44,55,65,55,33,67,78,54,55,55,44 Mode is 55
b. 33,44,65,55,33,67,78,54,55,55,44 Mode is 33, 44
c. 1,2,3,4,5,6,7,8,9 Mode is 1,2,3,4,5,6,7,8,9 or no
mode

58
2. Determine the mode of the following math test scores.

Test scores Frequency


25-30 4
31-36 7
37-42 3
43-48 5
49-54 8
55-60 12
61-66 6
N = 45

Modal class is: 56-60


12−8
Mode = 55.5 + (( )(6)
12−8)+(12−6)

Mode = 57.9

59
LESSON 3 : MEASURES OF
VARIATION/DISPERSION
Measures of variation is used to describe the distribution of the data. How
is the data distributed? Is it cluster in one area or is it really spread out?

Different measurements of variation

∙ Range
∙ Mean absolute deviation
∙ Variance
∙ Standard deviation

RANGE - The range is the simplest measure of variation to find.

It is simply the highest value minus the lowest value.

RANGE = MAXIMUM - MINIMUM

Since the range only uses the largest and smallest values, it is greatly
affected by extreme values, that is - it is not resistant to change.

Example

1. Find the range of the following ages of


mountaineers:
23,17,15,45,55,37,44,60

Range = 60-23 = 37

Interpretation: the ages of mountaineers are slightly spread out. Their


ages have big variation.

60
MEAN ABSOLUTE DEVIATION

∙ Mean absolute deviation is the average of how much the data values
differ from the mean.
∙ A small MAD value indicates a clustered data values.
∙ A big MAD value indicates a spread out data values

Example

1. Find and interpret the mean absolute deviation of the test


scores obtained by these two students.

Anna’s test scores Joy’s test scores


95 98
93 95
95 90
96 95
95 95

95+93+95+96+
Anna Mean = 95 = 94.8
5

(
Anna MAD = 94.8−95)+(94.8−93)+(94.8−95)+(94.8−96)+(94 = 0.72
.8−95) 5

98+95+90+95+
Joy Mean = 95 = 94.6
5

61
(
Joy MAD = 94.6−98)+(94.6−95)+(94.6−90)+(94.6−95)+(94 = 1.84
.6−95) 5

Interpretation:
Anna has a smaller MAD value than Joy which means Anna is more
consistent in her test results.

VARIANCE AND STANDARD DEVIATION

∙ Standard deviation is used to quantify the amount of variation or


dispersion of a set of data values and represented by the Greek letter
sigma σ or the Latin letter s. A low standard deviation is an indicator
that the data points tend to be close to the mean while a higher value
indicates a widerspread from the mean.
∙ The variance (represented by the Greek letter sigma that is squared or
the letter s2 ) is the square of the deviation of data sets from its
mean.It is use to determine how far or clustered a random data points
from their average value.

62
Example

1. Determine and interpret the variance and standard deviation of the


following number of votes obtained by each party list.

Partylist A Votes
Miguel 26
Dom 33
Ann 29
Gab 24
Hazel 30
Cherry 35

Partylist B Votes
Arman 35
Daniel 26
Elise 24
Dona 24
Mary 30
John 35

Solution:

Partylist A Mean = 35.4

Partylist A variance = 58.872

Partylist A Standard Deviation = 7.67

Partylist B Mean = 34.8

Partylist B variance = 66.768

Partylist B Standard Deviation = 8.17

Interpretation: Partylist A has a smaller standard deviation, therefore, they have a


higher chance to win or lose together than Party List B.

63
LESSON 4. QUARTILE, DECILE AND PERCENTILE
A measure of position determines the position of a single value in relation
to other values in a sample or a population data set. We commonly refer
to these measure of position as quantiles or fractiles.

Quantiles

It is a score distribution where the scores are divided into different

equal parts. There are three kinds of quantiles: Quartile, Decile and

Percentile.

Quartile- A measure of position that divides the ordered observations


or score distribution into 4 equal parts.

Decile- A measure of position that divides the ordered observations or score


distribution into 10 equal parts.

Percentiles - A measure of position that divides the ordered


observations or score distribution into 100 equal parts.

Formula for Quantile of Ungrouped Data

64
Example 1. A classroom teacher gave a quiz to 9 students. The scores
obtained are as follows:

10 5 9 4 2 6 3 4 8

Find the following:

a. 2rd Quartile
b. 7th Decile
c. 94th Percentile

Solution:

a. 2nd Quartile

2 3 4 4 5 6 8 9 10

Q Q Q
1 2 3

ANSWER: 5

b. 7th Decile

2 3 4 4 5 6 8 9 10

D D D D D D D D D
1 2 3 4 5 6 7 8 9

ANSWER:8

c. 94th Percentile

2 3 4 4 5 6 8 9 10

P P P P P P P P P
10 20 30 40 50 60 70 80 90

ANSWER:10

65
Interpretation:

a.Fifty percent of the data falls below and above the value of 5.

b.70 % of data falls below the value 8 and while 30 % of students scored

higher than 8 Example 2. Calculate the 3rd quartile. (refer to the formula of

grouped data)
Math 4 6 10 13 17
Score(x)
Frequency(f) 3 5 6 4 2

<cf 3 8 14 18 20

66
67
68
69
70
71
72
73
74
LESSON 5. THE NORMAL DISTRIBUTION
The Normal Distribution

Standard deviation is a statistic that characterizes a distribution of


score. It increases indirect proportion as the scores spread out more widely,
the larger the standard deviation, wider the spread of scores.

The meaning of standard deviation is best defined by normal


distribution of scores. The normal distribution is illustrated by the normal
curve. Normal curve is a symmetrical curve having a bell-like shape. The total
area under the normal curve is equal to 1, and represent all of the scores in a
normal distribution. In such a curve, the mean, the median, the mode are
identical or equal, so the mean falls at the exact center of the curve. The
curve has no boundaries in either direction, for the curve never touches the
baseline no matter how far it is extended. The curve is a curve of probability,
not certainty.

Figure 2.1 Percentage under the Normal Curve

75
The formula for the standard normal distribution is

−𝑧2
𝑦= 𝑒 2 where z = standard score, 𝑒 ≈ 2.1718.. and 𝜋
2𝜋
√ ≈ 3.14159..

Since the curve is symmetrical, this holds true for both side of
the mean. As presented in Figure 1, approximately 68.26% of the
scores lie between +𝜇 + 1𝜎 and -
𝜇 + 1𝜎. Furthermore, about 13.6% of the scores fall between 𝜇 + 1𝜎
and 𝜇 + 2𝜎. All of the scores in a normal distribution lie between the
mean plus or minus standard deviations.

If a set of scores is normally distributed, one can interpret any


particular score if he knows how far, in standard deviation units, it is
from the mean.
Example 1.The mean of the normal distribution is 50 and standard
deviation is 10.

a. How does an individual’s score of 60 compare with all the


other scores?
b. How about the score of 30?

Illustration 1.

8 19 30 41 52 63

Figure 2.2 A Normal Distribution of Score

76
a. If a person’s score is 60, then slightly more than 84. 135% of all the
other scores
in a distribution lie below his scores.
b. If a person’s score is 30, then slightly more than 97. 725% of all the
scores in a distribution fall above his scores.

Practical Application of Normal Curve

In the field of educational research, there are a number of practical


applications of the normal curve, among which are:

1. To calculate the percentile rank of scores in a distribution,


2. To normalize a frequency distribution; which is an important
process in standardizing a psychological test or inventory
3. To test the significance of observe measure in experiments, relating
them to the chance fluctuations or errors that an inherent in the
process and sampling and generating about population form which the
samples are drawn.

Standard Scores use a common scale to indicate how an individual compare to


other individuals in a group. These scores are particularly helpful in
comparing an individual’s relative position on different instruments.

Two most frequently used Standard Scores

Z-SCORES. These standard scores tell how far a raw score is from the mean
in standard deviation units. The formula is

𝑧= 𝑥−𝑥̅ or 𝑧 = 𝑥−𝜇
𝑠 𝜎
Where x- raw score

𝑥̅- mean

𝑠− standard deviation

T-SCORES. These are z-scores that are expressed in another way. The
formula is

𝑥−𝑥̅
𝑡 = 50 + 10 ( ) or 𝑡 = 50 + 10𝑧
𝑠

77
Example 2.

1. Jerry got a grade of 85 on the final examination in Linguistic for which the
mean grade was 75 and the standard deviation was 10. On the final
examination in Trigonometry for which the mean grade was 80 and the
standard deviation was 24, he received a grade of
92. In which subject was his relative standing higher?

Given. Linguistic: 𝑥̅ = 75, 𝑠 = 10 and 𝑥 = 89 and Trigonometry: 𝑥̅ = 80, 𝑠 = 17


and
𝑥 = 93.

z-scores t-scores

Linguistics
𝑧 = 𝑥−𝑥̅ = 89−75 = 1.4 𝑡 = 50 + 10𝑧 = 50 + 10(1.4) = 64
𝑠 10

Trigonometry
𝑧 = 𝑥−𝑥̅ = 93−80=0.77
𝑠 17 𝑡 = 50 + 10𝑧 = 50 + 10(0.77) = 57.7

Interpretation:

The score of Jerry in Linguistic is 1-unit standard deviation above the


mean. His score in Trigonometry is 0.5-unit standard deviation above
the mean. Thus, his relative standing was higher in Linguistic.

78
2. Professor Elise wanted to get a student’s equally weighted mean
achievement in
Algebra test and Geometry test. The data are shown below.
Course Test Score Mean Highest Standard
possible Deviation
score
Algebra 40 50 70 5

Geometry 95 125 200 20

Solution

Algebra test z score


̅ 𝑥 − 𝑥 40 − 50
𝑧= = = −2.0
𝑠 5

Geometry test z score


̅ 𝑥 − 𝑥 95 − 125
𝑧= = = −1.5
𝑠 20

Mean of z {(- 2.0) + (- 1.5)} / 2


score = - 1.75

Interpretation: The mean standard score of – 1.75 indicates that on an


equally weighted mean, the performance of the student was fairly
consistent that is - 2.0 standard scores below the mean in Algebra
and - 1.50 standard scores below the mean in Geometry.

79
The Normal Curve Areas

∙ Computing Normal Probabilities


∙ There are several different situations that can arise when
asked to find normal probabilities.

80
This can be shortened into two rules.

1. If there is only one z score given, use 0.5 for the second area,
otherwise look up both z scores in the table.
2. If the two numbers are the same sign, then subtract; if they are
different signs, then add. If there is only one z score, then use the
inequality to determine the second sign (< is negative, and > is
positive).

Example. If a random variable has the standard normal distribution,


what are the probabilities that it will take on a value:

3. Less than 1.25


4. Between 0.49 and 1. 25
3. 3. Between -0.75 and 1.45
4. Greater than or right of 2.67
5. Less than or left of 0.75
81
82
Application of the Normal Curve

You are now ready to solve normal distribution problems. In doing so,
you must always bear in mind the difference between continuous
random variable and discrete random variables. Each category has its
own way of calculating the areas under their standard normal
distribution.

Continuous random Variable (Measurable)

Example 1. The scores of the grade 6 pupils have a mean of 5.35 and
standard deviation of 0.15.

a. What percentage of all these scores are lower than 4?


b. What percentage of these scores are between 5 and 6?

Solutions

c. Lower than 4.
Given 𝜇 = 5.35, 𝜎 = 0.15 and 𝑥 = 4

The area between 0 and is .


Hence, the percentage of all these scores are lower than 6 is %.

b. Area between 5 and 6

83
Area between 5 and 6 = Area between 5 and 5.35 + Area
between 5. 23 and 6.
=___________+ _________
= _________or _____ %

Thus, percentage of these scores are between 5 and 6 is %.

Discrete Random Variable(Countable)

Example 2. Princess Joy is a Home Economics teacher. She knows


from experience that the number of budget meals she sells each day
is a random variable having approximately a normal distribution with
the mean equal to 43.5 and standard deviation equal to 5.6. What are
the probabilities that in any value day she will sell?

a. Exactly 30 budget meals;


b. At most 30 budget meals;
c. At least 30 budget meals.

Solution:

Because the data are discrete, the values of this discrete random
variable shall be spread over a continuous scale by representing
each whole number k by interval form

𝑘− 1
to 𝑘+ 1
2 2

84
In this example, 𝑘 = 30,

𝑥 = 𝑘 − 21= 30 − 1
2
= 29.5 𝑥2= 𝑘 − 2 = 30 + 12= 30.5
1 1

a. Exactly 30 budget meals


Area between 29.5 to 43.5 Area between 43.5 to 30.5
𝑥−𝜇 𝑥−𝜇
𝑧1 = 𝜎 𝑧2 = 𝜎

The area is . The area is .


Illustration:

The area between 29.5 and 30.5 (exactly 30) = 𝑧1−𝑧1= or %.

Hence, the probability that she will sell exactly 30 budget meals is__ %.

b. At most 30 budget meals.


𝑥−𝜇
𝑧= Illustration:
𝜎

85
The area is .

Area at most = 0.5 - _________


= ____ or __ %.

Thus, the probability that she will sell at most 30 budget


meals is ______ %.

86
87
88
89
90
LESSON 6 : CORRELATION AND LINEAR
REGRESSION
Correlation measures the association or the strength of the relationship
between two variables say x and y.

Definitions

⮚ Two variables are positively correlated if the values of the two


variables both increase.
⮚ Two variables are negatively correlated if the values of one variable
increase while the values of the other decrease.
⮚ Two variables are not correlated or they have zero correlation if one
variable neither increases nor decreases while the other increases.

Verbal Interpretation

The degree of correlation can be determined by correlation coefficient.


Its value represents an interpretation as shown in the table below.

r Verbal Interpretation

0.00 No correlation

±0.01 to ±0.20 Slight Correlation

±0.21 to ±0.40 Low Correlation

±0.41 to ±0.70 Moderate Correlation

±0.71 to ±0.80 High Correlation

±0.81 to ±0.99 Very High Correlation

±1.0 Perfect Correlation

91
Pearson Product- Moment Correlation(r)

⮚ The most familiar sort of statistical tool in quantifying the


linear relationship between two random variables, x and y.
⮚ Data are parametric( numerical measurement describing a
characteristic of a sample).

Formula

Steps in Solving Correlation

1. State the Null Hypothesis (Ho) and the Alternative Hypothesis ( Ha).
2. Determine the tabular value (TV), degree of freedom(df) = N-2.
3. Determined computed value (CV).
4. State the conclusion.
a. Decision: i. rc>rt (means Reject Ho) and ii. rc<rt (means Accept Ho).

Example:

1. Calculate and analyze the correlation coefficient between the number


of study hours and the number of sleeping hours of different
students at 0.05 level of significance.

Number of 2 4 6 8 10
Study
Hours (x)
Number of 10 9 8 7 6
Sleeping
Hours
(y)
92
Solution:

1. Ho: There is no significant relationship between the number of study


hours and the number of sleeping hours of different students.
Ha: There is significant relationship between the number of study
hours and the number of sleeping hours of different students.
2. Tabular Value: 𝛼 = 0.05 and df= N-2= 5-2=3
(3, 0.05)=0.878

3. Computed Value:

Student 𝑥 𝑦 𝑥∙𝑦 𝑦2
𝑥2

1 2 10 20 4 100

2 4 9 36 16 81

3 6 8 48 36 64

4 8 7 56 48 49

5 10 6 60 100 36

∑𝑥 ∑𝑦 ∑ 𝑥𝑦 ∑ 𝑥2 ∑ 𝑦2
𝑁=5
= 30 = 40 = 220 = 220 = 330

4. Conclusion

Based on the result of 𝑟 = −1 which is less than the tabulated value of


0.878, Do not reject Ho. This implies that there is no significant
relationship between the number

93
of study hours and the number of sleeping hours of different students. The
result of r also implies perfect correlation.

Linear Regression

Regression Analysis is very powerful tool in the field of statistical analysis


specifically in predicting the value of one variable to the given value of
another variable, and those variables that are related to each other.
Therefore, it is used when predicting the behavior of a variable. The
regression equation explains the amount of variations visible in the
independent variable x. It is actually an equation of a straight line.

The purpose of regression is to determine the trend of the two


variables as related to each other whether the trend is rising or
falling.

Formula:

𝒚 = 𝒂 + 𝒃𝒙

where:

y= criterion measure,

x= predictor,

a= ordinate or the point where the regression line crosses the y-


axis, and

b= beta weight or the slope of the line

94
To get the regression equation, the value of a and b are computed
using the
formula below.

(∑ 𝒚)( ∑ 𝒙𝟐) − (∑ 𝒙)(∑ 𝒙𝒚)


𝒂=
𝒏(∑ 𝒙𝟐) − (∑ 𝒙)𝟐

and

𝒏(∑ 𝒙𝒚) − ∑ 𝒙 ∑ 𝒚
𝒃=
𝒏(∑ 𝒙𝟐) − (∑ 𝒙)𝟐

where:

n= number of pairs

Example1.

The data in the table represent the membership at a university


mathematics club during the past 5 years.

Number of Years (x) Membership (y)

1 25

2 30

3 32

4 45

5 50

95
Form a curve of the form𝒚 = 𝒂 + 𝒃𝒙to predict the membership 5
years from now.

Solution:

x y 𝒙𝟐 𝒙𝒚

1 25 1 25

2 30 4 60

3 32 9 96

4 45 16 180

5 50 25 250

∑ 𝒙 = 𝟏𝟓 ∑ 𝒚 = 𝟏𝟖𝟐 ∑ 𝒙𝟐 = 𝟓𝟓 ∑ 𝒙𝒚 = 𝟔𝟏𝟏

Since you need to predict the membership five years from now, or at year 10,
substitute 10 for x in the equation. Thus, 5 years from now, 𝐲 = 𝟏𝟔. 𝟗 + 𝟔.
𝟓(𝟏𝟎) = 𝟖𝟏. 𝟗 ≈ 𝟖𝟐.

Therefore, five years from now, the club would have 82 members.

96
97
98
99
CHAPTER MATHEMATICS AS A TOOL :

5 CODES AND CRYPTOGRAPHY

LEARNING OUTCOMES:

At the end of the chapter the students are expected


to :

∙ Perform the coding of a word text using the


different method
∙ Decrypt the different message.
LESSON 1 : CODING THEORY
Introduction

Communication systems and storage devices nowadays are not reliable


as it seems. The communication system is usually composed of sender (or
message source), communication channel, and the receiver. Communication
channel is the physical medium through which information is transmitted.
Some examples of such channels are telephone lines, internet cables, fiber-
optic lines, and air. Storage of data can be also be considered as channels,
like hard drives, CD-ROMs, DVDs, and the likes.

As the message travels from the sender to the receiver through the
channel, it encounters several “noises” that alters the message in the channel
that will cause disruption and errors in the messages. Examples of noises are
caused by sunspots, lightning, meteor showers, or even human errors like
poor typing and poor hearing.

One of the tasks of coding theory is to detect, or even correct, those


errors. In transmitting messages, coding is defined as source coding and
channel coding. There are

100
two processes in coding; encoding and decoding. Encoding is transforming
messages into bits of message that is suitable to the communication channel
while decoding is the opposite process of encoding.

Source Coding

Receiver
Sender Noise

Source Encoder Communication Channel Source Decoder

Source coding is defined as converting the messages from the sender


into bits suitable to the communication channel. An example of this is the
ASCII code that converts each character in the message to a byte of 8 bits.
The commonly used alphabet is the binary system where it uses the numbers
0 and 1.

Example 1.a. Consider the source encoding of four directions north, south,
east, and west as follows:

north 00 , south 01 , east 10 , west 11 .

Suppose the message “north”, which is encoded as 00, is transmitted


over a noisy channel. The message may encounter errors and may be
received as 01. The receiver will get the message bit 01 and decode it as
“south” without realizing that the message is corrupted.

101
With this problem at hand, additional process of encoding is
required.

Channel Coding

Receiver
Sender

Source Encoder Noise SourceDecoder

Channel Encoder Communication Channel Channel Decoder

Channel coding is defined as adding some form of redundancy to the


source encoded message so that the errors can be detected or even
corrected.

Parity Check

Example 1.b. Consider the Example 1.a, we perform the channel encoding by
adding a redundancy bit of size 1 as follows:

00 000 , 01 011 , 10 101 , 11 110 .

Suppose again that the message “north”, which is source encoded as


00 and again channel encoded as 000, is transmitted over a noisy channel.
Suppose that the message encounter only one error in the communication
channel. Then the received word can be any of the following messages; 001,
010, or 100. In this manner, we can detect that there is an error in the
message since none of the three possible messages is among the source
encoded messages.

This encoding scheme is called parity check, where in a single bit is


added to the message as redundancy bit. A bit string is said to have an odd
parity if there is an odd

102
number of 1s, and even parity if there is an even number of 1s. We add
redundancy bit to message such that it will become an even parity.

In this encoding scheme, we can only detect if the messages received


encountered a disruption because of errors or noises, however it does not allows
us to correct them. For example, if we receive 100, we do not know if it is from
000, 101, or 110. However, adding more redundancy will allow us to correct
errors.

Example 1.c. Consider the Example 1.a, we perform the channel encoding by
adding a redundancy bit of size 3 as follows:

00 00000 , 01 01111 , 10 10110 , 11


11001 .

Same with Example 1.b, the message “north” will be encoded up to the
message 00000. Because of the noises in the channel, we say that only one error
is introduced to the message that will give us five different possible messages
10000, 01000, 00100, 00010, or 00001. Assume that 10000 is received. Then we
can be sure that 10000 is from the correct message 00000 since if we are going
to compare 10000 to the other encoded messages such as 01111, 10110, and
11001, there will be at least two errors in the message.

Note that in this encoding scheme that can correct errors, information
transmission speed will be at cost since we have to transmit 5-bit message for a
message of a size of 2 bits.

Repetition Code

Suppose that the source encoding is already done and that the encoded
message is of fix length k. Then the channel encoding by repetition is performed
by taking the k bits, then repeat it 2r + 1times, where r ≥ 1 is a fixed integer.

Example 2.a. Suppose that the source encoded message is 101 where k = 3. Then
choose = 2 , so we must repeat 101, 5 times. This will give us 101101101101101.

Assume that the message is transmitted through a noisy channel and distorted.
The received message is 111101100011100.

The decoding process will be done in this manner. Consider the positions 1,
4, 7, 10, 13 of the received message. We will take the most frequent bit as the
first decoded
103
bit. Then, consider 2, 5, 8, 11, 14 positions and do the same process to get
the second bit. For the third and last bit, consider the positions 3, 6, 9, 12,
and 15. With this, the decoded message will be 101.

In this encoding scheme, we can detect and correct more than one
error for the cost of transmission speed of the messages since like the
example above, you will transmit 15-bit message for a message of size of 3
bits.

Modular Arithmetic

Congruence

Let a, and b are integers and m is a natural number. We say “a


is congruent to b modulo m” in symbols a ≡ b (mod m) , if m divides a -
b or if m divides b - a. In other words, the difference of a and b is
divisible by m. In addition, if m > 0, and r is the remainder when b is
divided by m then r ≡ b (mod m) . The integer r is called the least
residue of b (mod m) .

Examples

I. Verify if the following congruence are true.


1.) 3 ≡ 9 (mod 2)
2.) 4 ≡ 15 (mod 3)
3.) 7 ≡ 15 (mod 4)
4.) 5 ≡ 20 (mod 3)
5.) 4 ≡ 20 (mod 3)
Answers:
1.) It is true since 9 - 3 = 6, which is divisible by 2.
2.) It is false since 15 - 4 = 11, which is not divisible by 3.
3.) It is true since 15 - 7 = 8, which is divisible by 4.
4.) It is true since 20 - 5 = 15, which is divisible by 3.
5.) It is false since 20 - 4 = 16, which is not divisible by 3.

104
II.Solve the least residue of the following.
1.) 29 (mod 3)
2.) 35 (mod 4)
3.) 50 (mod 7)
4.) 30 (mod 4)
5.) 25 (mod 5)

Answers:
1.) The answer is 2, since 2 is the remainder when 29 is
divided by 3.
2.) The answer is 3, since 3 is the remainder when 35 is
divided by 4.
3.) The answer is 1, since 1 is the remainder when 50 is
divided by 7.
4.) The answer is 2, since 2 is the remainder when 30 is
divided by 4.
5.) The answer is 0, since there is no remainder when 25
is divided by 5.

III.Solve the following modular arithmetic.


1.) 10 − 4 (mod 5)
2.) 11 + 7 (mod 3)
3.) 8 + 7 (mod 7)
4.) 20 − 7 (mod 5)
5.) 31 − 6 (mod 4)
Answer
s:
1. 1
2. 0
3. 1
4. 3
5. 3

105
Check Digits and Check Codes

There are several methods in producing identification numbers which


are unique. In the following methods, modular arithmetic is used to produce
and verify identification numbers. The examples are; the Universal Product
Code (UPC), United States Postal Services (USPS), the Credit Card, and the
International Standard Book Number. Each examples uses their last digit as
the check digits to verify the identification number.
Universal Product Code (UPC), is mainly used in products sold in
department stores and groceries. The UPC consists of barcodes with 12 digits
where the last one is the check digit. The International Standard Book
Number (ISBN) is used on books where usually found at the last page of the
book. ISBN can be ISBN-10 or ISBN-13 where they used 10 digits or 13 digits
string of number respectively with the last digit as the check digit. Lastly, the
United States Postal Services (USPS) is consist of 11 digits, while the Credit
Card uses 16 digits where both of them uses their last digits as the check
digits.

The table below summarize the formulas for the check digits.

106
d =9− d +d +d +d +d +d +d +d +d +d
Check 11 ( 1 2 3 4 5 6 7 8 9 10 )(mod 9)
Digit
d
11

Credit Card
d = 10 − 2d + d + 2d + d + 2d + d + 2d + d + 2d + d + 2d + d + 2d + d + 2d
Check 16 ( 1 2 3 4 5 6 7 8 9 10 11 12 13 14 15 ) (mod10)

Digit Note: Add all the digits, treating the two-digit numbers as
d two single digits.
16

Examples:

1. Determine the check digit for a new product given that the first 11 digits
are 8-21345- 67132- ?

Solution:

Let d
1 2
= 8, d = 2, d
3 = 1, d
4 = 3, d
5 = 4, d
6 = 5, d
7 = 6, d
8 9
= 7, d = 1, d
10 = 3, and d
11 = 2.

Then,
d12 = 10 − (3(8) + 2 + 3(1) + 3 + 3(4) + 5 + 3(6) + 7 + 3(1) + 3 + 3(2))(mod10)
d12 = 10 − 86 mod10
d12 = 10 − 6
d12 = 4

Hence, the check digit is 4.

2. Determine the check digit of book with an ISBN-10 first 9 digits of 0-


17-316444- ?

Solution:

Let d
1 2
= 0, d = 1, d
3 = 7, d
4 = 3, d
5 = 1, d
6 = 6, d
7 = 4, d
8 = 4,and d
9 = 4.

107
Then,
d10 = 11 − (10 (0) + 9 (1) + 8(7) + 7 (3) + 6 (1) + 5(6) + 4 (4) + 3(4) + 2 (4))(mod11)
(
d = 11 −158 mod11
10 )
d10 = 11 − 4
d10 = 7
Hence, the check digit is 7.

3. Verify if the given Credit Card identification number 4301-1234-8888-3751


is valid.

Solution:

1 = 4, d2 = 3, d3 = 0, d4
Let d = 1, d
5 6
= 1, d
7
= 2, d
8
= 3, d
9
= 4, d
10
= 8, d
11
= 8, d
12
= 8, d
13
= 8, d =

14 = 7,and d15 = 5.
3, d

It is given that the check digit is 1. We will verify it with the formula.

( ) + 3 + 2 (0) + 1 + 2 (1) + 2 + 2 (3) + 4


d = 10 − (2 4
16

()
+2 8 +8+2 8
( ) + 8 + 2 (3) + 7 + 2 (5)) (mod10)
d16 = 10 − (8 + 3 + 0 + 1 + 2 + 2 + 6 + 4
(
+ 16 + 8 + 16 + 8 + 6 + 7 + 10) mod10
)
d16 = 10 − (8 + 3 + 0 + 1 + 2 + 2 + 6 + 4
(
+ 1 + 6 + 8 + 1 + 6 + 8 + 6 + 7 + 1 + 0) mod10
)
( )(mod10)
d = 10 − 70
16

d16 = 10 − 0

d16 = 1 + 0

d16 = 1

Hence, the check digit is correct and the identification number is valid.

108
109
110
111
112
113
114
115
116
117
118
119
LESSON 2 : CRYPTOGRAPHY
∙ Originated from two Greek words such as KRYPTO which means
hidden and GRAPHENE means writing.
∙ Is a method of making and breaking of secret codes.
∙ It the method that is commonly used in military, some government
agency’s transaction, business firms such as bank, insurance, etc. to
secure secrecy of the information.
∙ It uses two processes such as encryption and decryption.

Example:

(Plain text) Maths is fun (Coded Form) PDWKV LV IXQ

ENCRYPTION

∙ Is the process of transforming plain text into code form using a certain
algorithm

DECRYPTION

∙ Is the process of returning/converting back the coded message into plain text.

** PLAIN TEXT = refers to the original message

** CIPHERTEXT = refers to the coded message

KEY = refers to the strings of information that is used to reveal the


encrypted message into readable form.

Example:

120
Note : Every letter of the English Alphabet should be paired to at least one character of
the Coded Alphabet.

Simple Methods of Cryptography

SHIFT CIPHER (CEASAR CIPHER) = it is the simple type of substitution cipher. It uses
shift in forming the key of cryptography. The cipher text is obtained from taking an
equivalent of a single letter of the alphabet to another letter by doing a uniform number of
shifts either left or right. Each letter of the English alphabet should be matched exactly to
one letter of the cipher alphabet.

Illustrations:

Using a shift of 3 to the left (the commonly used number of shifts)

A B C D E F G H I J K L M N O P Q R S T U V W X YZ

D E F G H I J K L MNO P Q R S T U V W X Y Z A BC

( http://en.wikipedia.org/w/index.php?title=File:Caesar3.svg&page=1)

Using a shift of 5 to the left,

A B C D E F G H I J K L M N O P Q R S T U V W X YZ

V W X Y Z A B C D E F G H I J K L M N O P Q R S TU

Take note : 1st line of letters are the English Alphabet while the letters on the 2nd row are
the equivalent cipher character/alphabet

Examples:

1. Using the key of shift of 3 to the right, encrypt the Word “ CRYPTOGRAPHY”

CRYPTOGRAPHY FUBSWRJUDKCT

121
2. Decrypt the cipher text, “BJY DN BJJY”, using the key of 5-shifts to the left.

“BJY DN BJJY” God is good.

Using a Modulo Operator

The sender of the message uses the key K to encrypt and to decrypt the
secret message. The key K may have any integer value from 0 – 25 and this
will be shared to the person/s who will be receiving the secret message.

To encrypt the message:

1.Express the letters of the alphabet into an integer from 0 to 25, that matches
its order, for example A = 0, B = 1, C = 3 ….., then label them as C

2.Calculate Y = (C+ K) mod 26 , for every letter of the message.

3.Convert the number Y into a letter following the order of the letter of the
alphabet.

Illustration: Encrypt the message “MMW is fun to learn”

Let K = 5

Step 1.

M = 12 , W = 22, I = 8, S = 18, F = 5, U = 20, N = 13, T = 19, O = 14, L =


11,

E = 4, A = 0, R = 17
122
Step 2.

M M W I S F U N T O L E A R N
12 12 22 8 18 5 20 13 19 14 11 4 0 17 13
+ 5 5 5 5 5 5 5 5 5 5 5 5 5 5
5

17 17 27 13 23 10 25 18 24 19 16 9 5 22 18
Y = (17 17 27 13 23 10 25 18 24 1 16 9 5 22 18 mod26
9 )
Y = 17 17 1 13 23 10 25 18 2 19 16 9 5 22 18
4

Step 3.

RRB NX KZS YT QJFWS ENCRYPTED MESSAGE

To Decrypt the Message:

1.Express the letters of the alphabet into an integer from 0 to 25, that matches
its order, for example A = 0, B = 1, C = 3 ….., then label them as Y

2.Calculate C = (Y - K) mod 26 , for every letter of the decrypted message.

3.Convert the number C into a letter following the order of the letter of the
alphabet.

123
Illustration: Decrypt the cipher text RRB NX KZS YT QJFWS

Let K = 5

Step 1

R = 17 , B = 1, N = 13, X = 23, K = 10, Z = 25, S = 18, Y = 24, T = 19, Q = 16,

J = 9 , F = 5, W = 22,

Step 2

R R B N X K Z S Y T Q J F W
S
17 17 1 13 23 10 25 18 24 19 16 9 5 22
18
-5 5 5 5 5 5 5 5 5 5 5 5 5 5 5

12 12 - 4 8 18 5 20 13 19 14 11 4 0 17 13
C = ( 12 12 - 4 8 18 5 20 13 19 14 11 4 0 17
13) mod 26
C = 12 12 22
8 18 5 20 13 19 14 11 4 0
17 13

Step 3

MMW IS FUN TO LEARN DECRYPTED MESSAGE

3. Encrypt the word text “FATIMA” using the Modulo Operator, given k = 20

124
Step 1

F = 5, A = 0, T = 19, I = 8, M = 12

Step 2
F A T I M A

5 0 19 8 12 0

+ 20 20 20 20 20 20

25 20 39 28 32 20

Y = (25 20 39 28 32 20) mod 26

Y = 25 20 13 2 6 20

Step 3

ZUNCGU ENCRYPTED MESSAGE

Review: Arithmetic Modulo (Modular Arithmetic)

Modular Arithmetic

= also known Clock Arithmetic

= it is an operator (mod), which seeks for a remainder when two numbers are
divided.

= it is denoted by the symbol k mod M = r, (which read as k modulo M equals r)

125
How to Calculate the Arithmetic Modulo?

A. Let k and M are any positive Integers

∙ Divide k by the value of M to obtain the quotient (q) and the remainder (r)
∙ Such that k = Mq + r , 0 ≤ r < M

Examples:

1. Find 15 mod 6.

Solution :
15 ÷ 6 = 2 remainder 3 , hence , 15 mod 6 = 3

Checking : k = Mq + 3, where k = 15, M = 6, q = 2, and r =


3

15 = 6 * 2 + 3

15 = 12 + 3

15 = 15
2. Solve 8 mod 10.

Solution:

8 ÷ 10 = 0 remainder 8, hence, 8 mode 10 = 8

Checking : k = Mq + 3, where k = 8, M = 10, q = 0, and r = 8

8 = 10* 0 + 8

8=8

B. Let k be any negative integer and M is positive integer.

∙ Divide /k/ by the value of M to obtained the r’, (r’ ≠ 0)


∙ Therefore, r = M – r’.

126
Example:

Evaluate – 6 mod 4.

Solution:

/- 6/ ÷ 4 = 1 remainder 2, r’ = 2

r=4–2=2 , hence - 6 mod 4 = 2

127
128
129
130
131
CHAPTER MATHEMATICS AS A TOOL :

6 APPORTIONMENT AND VOTING

LEARNING OUTCOMES:

In this lesson, the students are expected to:


∙ Solve apportionment problems involving the
different method.
∙ Differentiate : Hamilton apportionment
method, Jefferson apportionment method and
Huntington – hill method.
∙ Conduct voting using different methods.

LESSON 1 : APPORTIONMENT

⮚ Is a method of dividing a whole into various parts.

⮚ The process originated in 1790 in the U.S. congress.

⮚ They want to established or select a fairly number of


representatives of each state based on state population in the
U.S. congressional seat.

⮚ It means that the numbers of representative ( the seat) is


proportion to the population size being represented.
⮚ Sometimes called “ the equal proportion “.

⮚ Different plans were introduce to select right numbers of


representative .

⮚ has expanded into different applications in the modern


world in economics, accounting , business, law , etc.

132
The Hamilton Plan

⮚ Early apportionment method used in US congress was


introduced by Alexander Hamilton

⮚ Given the number of seat in the US congress will be


apportioned between states proportionally to their
population.

⮚ Standard divisor (D) –the number of voters represented by


each representative.

⮚ Standard quota (Q) – the whole part of the quotient when


the population of the sub – group is divided by the standard
divisor.

𝑡𝑜𝑡𝑎𝑙 𝑝𝑜𝑝𝑢𝑙𝑎𝑡𝑖𝑜𝑛
Formulas: Standard Divisor
𝑛𝑢𝑚𝑏𝑒𝑟 𝑜𝑓 𝑟𝑒𝑝𝑟𝑒𝑠𝑒𝑛𝑡𝑎𝑡𝑖𝑣𝑒
(D)=
𝑁
D
𝑅

=
𝑠𝑢𝑏−𝑔𝑟𝑜𝑢𝑝 𝑝𝑜𝑝𝑢𝑙𝑎𝑡𝑖𝑜𝑛
Standard quota
𝑠𝑡𝑎𝑛𝑑𝑎𝑟𝑑 𝑑𝑖𝑣𝑖𝑠𝑜𝑟
(Q)=
𝑛
Q=
𝐷

Note : 1. Standard quota , Q must be an integer. In case of decimals,


just drop the decimal values .
2. When the total standard quota is not equal to given total
apportioned
or the number of representative , place an additional representative
to the next the sub – group representative with the highest decimal
value until the representatives are complete.

133
Example:

A new school offering the complete six grades in high school has the
following enrollment in the different grades below. The administration
are to apportioned the 20 teachers for each grade. Calculate
a. The standard divisor
b. The standard quota

Grades students
Grade 12 40
Grade 11 35
Grade 10 22
Grade 9 38
Grade 8 25
Grade 7 39
Total 199

Solution :
𝑁 19
a. Standard Divisor , D = =9 = 9.95
𝑅
20

𝑛 40
b. grade 12 Standard quota , Q = = = 4.02
12 𝐷 9.95

𝑛 35
grade 11 Standard quota , Q = = = 3.51
11 𝐷 9.95

𝑛 22
grade 10 Standard quota ,Q = = = 2.21
10 𝐷 9.95

134
𝒏
Grade student Q= Q Q Corrected
𝑫 no. of
teacher
s
Grade 12 40 40 4.02 4 4
9.95
Grade 11 35 35 3.51 3 4
9.95

Grade 10 22 22 2.21 2 2
9.95
Grade 9 38 38 3.81 3 4
9.95

Grade 8 25 25 2.51 2 2
9.95
Grade 7 39 39 3.91 3 4
9.95

Total : 199 17 20

Analysis :

The above example is already an application of apportionment. If the


given problem represents the states , voters and representatives
related to the origin of apportionment
,

Answer the following questions:

1. Which represent the different state in the problem ?

2. Which represent the different voters in the problem ?

3. Which represent the number of representatives in the problem ?

4. Define apportionment in your own words ?

135
The Jefferson Plan

⮚ Another apportionment method used in US congress was


introduced by Thomas Jefferson

⮚ This method uses a modified standard divisor the arrives at


the correct or exact numbers of representative using trial
and error .

⮚ The modified uses an assume value always smaller than the


standard divisor.

Example :

A new school offering the complete six grades in high school has the
following enrollment in the different grades below. The administration
are to apportioned he 20 teachers for each grade. Calculate the
number of teachers for each grade
using the Jefferson apportionment method.

Grades students
Grade 12 40
Grade 11 35
Grade 10 22
Grade 9 38
Grade 8 25
Grade 7 39
Total 199

136
Solution
:
a assume values for Modified Standard divisor ( D )
m
.

say , D = 9
m

b. by trial and error, solve the number of


teachers
𝑛 40
grade 12 Standard quota , Q = = = 4.44
12 𝐷 9

𝑛 35
grade 11 Standard quota ,Q = = = 3.88
11 𝐷 9

𝑛 22
grade 10 Standard quota , Q = 𝐷
= 9
= 2.44
10

note: grade 12 supposedly must have 4 teachers.

c. Another assumption and trial.

say ,D = 8.7
m

𝑛 40
grade 12 Standard quota , Q = = = 4.59
12 𝐷 8.7

𝑛 35
grade 11 Standard quota , Q = = 8.7 = 4.02
11 𝐷

𝑛 22
grade 10 Standard quota , Q = 𝐷
= 8.7 = 2.52
10

137
𝒏
Grade student Q= Q correct
𝑫 no. of
teacher
s
Grade 12 40 40 4.59 4
8.7

Grade 11 35 35 4.02 4
8.7

Grade 10 22 22 2.52 2
8.7

Grade 9 38 38 4.36 4
8.7

Grade 8 25 25 2.87 2
8.7

Grade 7 39 39 4.48 4
8.7

Total : 199 20

Analysis :

The above problem , the number of apportionment is exact


using the modified standard divisor.

Apportionment principle

⮚ A new representative is added to a sub – group due to


an increase in population.

⮚ The representative is assigned to the group in such a


way it gives the smallest relative unfairness of
apportionment.

138
𝑨
Formula : R =
𝑪

Where : R = relative unfairness of apportionment


1 - C2 /
A = absolute unfairness of apportionment = / C

C = average population of the sub – group receiving the new


Representative.
𝑠𝑢𝑏−𝑔𝑟𝑜𝑢𝑝
C=
𝑛𝑜.𝑜𝑓 𝑟𝑒𝑝𝑟𝑒𝑠𝑒𝑛𝑡𝑎𝑡𝑖𝑣𝑒

Example:

RBSN company wants to add a new call center agent in one of its
office. Report indicate an increase in the daily calls of the offices in the
past month. Determine which office should get the additional agent.
Use the apportionment Principle to justify your answer.

Office branch Number of agents Ave. no. of call / day


Makati 62 882
Ortigas 48 996

Solution

𝑨
Office C C A=/C –C / R=
1 2 `1 2
branch 𝑪

882 996
Makati = 14.00 = 20.75 6.75 0.48
63 48

882 996
Ortigas = 14.22 = 20.32 6.10 0.30
62 49

139
Answer. R = 0.30 (the lowest ) means the new agent or
representative will goes to Ortigas office.

Analysis.

1 The additional one in C and C indicates that we assume that office or


1 2
. state
receives the additional agent or representative.
2 C and C with no additional indicates that office or state does not
1 2
. receives
the additional agent or representative.
3. Repeat same procedure when adding another new
representative is apportioned.

Huntington – Hill apportioned method

⮚ The present method of apportionment being use by the US


congress

⮚ The method that make use of equal proportion.

⮚ The new additional representative to a sub – group must


have the highest Huntington number.

(𝑃𝐴 )2
Formula : H=
𝑎 ( 𝑎+1 )

Where : PA = population of the sub – group


a = the current number of representative of sub
– group A
H = Huntington – Hill number

140
Example

RBSN company wants to add a new call center agent in one of its
office. Report indicate an increase in the daily calls of the offices in the
past month. Determine which office should get the additional agent.
Use the Huntington – Hill apportionment to justify your answer.

Office branch Number of agents Ave. no. of call / day


Makati 62 882
Ortigas 48 996

Solution

(𝑃𝐴 )2 882
H= = = 199.16
𝑎 ( 𝑎+1 ) 62 2( 62+1 )

(𝑃𝐴 )2 996
H= = 2 = 421.77
𝑎 ( 𝑎+1 ) 48 ( 48+1 )

∙ Since H = 421.77 has the highest Huntington – Hill number, So


that the Ortigas office will get the new agent.

141
142
143
144
145
LESSON 2 : VOTING METHODS
1. Plurality Method

∙ Each voter selects one candidate or choice.


∙ The winner is the candidate or choice with the most votes.

Example 1: If we vote on a coffee shop for the kind of drinks and 14


people vote for black coffee, 9 for mocha frappe, and 10 people for
tea, then the winner of the plurality election is black coffee.

Note:Black coffee did not win by a majority, because it has


fewer than 50% of the votes. Black coffee won by a
plurality.

Example 2: (Minimum Votes Needed to Win) There are 100 voters in


plurality elections between Duterte, Roxas, and Poe. After 70 votes
have been counted, Duterte has 38 votes, Roxas has 18 votes, and
Poe has 14 votes. How many remaining votes does Duterte need to
guarantee he wins?

How to solve this: First, pick your candidate’s biggest competition, in this
case Roxas. Pretend all 30 votes go to Roxas and Duterte. Let x be the
number of votes Duterte needs to tie Roxas in this scenario. Then
Roxas gets 30 – xof the remaining votes. Since it’s a tie, 38 + x = 18 + (30
– x). Solve for x to get x = 5. If Duterte gets more votes than this, he is
guaranteed to win, and so the answer is the smallest number bigger
than x, in this case 5 votes.

Strategic Voting: When a person votes in a way that does not reflect his
or her true preferences in an attempt to improve the outcome of the
election form that person’s point of view, it is called strategic voting.

146
Runoff Election:
· First a plurality vote is taken.
· If one candidate has more than 50% of the vote, that candidate wins.
· If no candidate has a majority of the votes, a second plurality election is held
with a designated number of the top candidates.
· This process repeats until one candidate has more than 50% of the votes.

Example 3: In May 2016 Presidential elections, Duterte with 16, 601, 997 votes,
Roxas with 9, 978, 175 votes, and Poe with 9, 100, 991 votes. If there had been
a runoff between Duterte and Roxas, what percentage of Poe’s supporters would
have needed to vote for Duterte for him to have a majority of the vote?

How to solve this: First, adding up the votes from all three candidates, we see
that there are 35, 681, 163 votes total. A majority is 1 more than half of this, so
1+ (35, 681, 163) = 17, 840, 582.5 or 17, 840, 583. Duterte already has 16, 601,
997 votes so he needs 17, 840, 583 – 16, 601, 997 = 1, 238, 586. This is

2. Borda Count Method


∙ Award points to candidates based on preference schedule, then
declare the winner to be the candidate with the most points.

Ballo Point
t s
B gets 4
1st points
D gets 3
2BndD points

147
In general, if N is the number of candidates…
∙ Each first-place vote is worth N points.
∙ Each second-place vote is worth N – 1points.
∙ Each third-place vote is worth N – 2 points.
∙ …
∙ Each Nth-place (i.e., last place) vote is worth
1 point.

Whichever candidate receives the most points wins


the election.

Example 1: A corporation would like to invite a new investor. The


possibilities are Ayala (A), Bonifacio (B), Calixto (C), and Dancel (D).

All investors of the said corporation are polled. The results:

Number of 35 30 20 15
Voters
1st choice B D C D

2nd choice C A A C

3rd choice A B B A

4th choice D C D B

Who is the winner under the Borda Count?

148
How to solve this:

A Borda Count Election

Number of Voters 35 30 20 15
1st choice (4 B: 140 D: 120 C: 80 D: 60
points)
2nd choice (3 C: 105 A: 90 A: 60 C: 45
points)
3rd choice (2 A: 70 B: 60 B: 40 A: 30
points)
4th choice (1 D: 35 C: 30 D: 20 B: 15
point)

Ayala (A): 70 + 90 + 60 + 30 = 250 points

Bonifacio (B): 140 + 60 + 40 + 15 = 255


points

Calixto (C): 70 + 30 + 80 + 45 = 225 points

Dancel (D): 35 + 120 + 20 + 60 = 235

points Bonifacio wins.

149
Example 2: A Borda Count Election is held between Theo,
Nicole, and Rafael.

Number of 10 8 7 5
Voters
1st choice Theo Nicole Nicole Rafae
l
2nd choice Nicole Rafael Theo Nicole
3rd choice Rafael Theo Rafae Theo
l

How to solve this:

Number of Voters 10 8 7 5
1 choice (3
st
Theo: 30 Nicole: Nicole: Rafael:
points) 24 21 15
2 choice (2
nd
Nicole: Rafael: Theo: 14 Nicole:
points) 20 16 10
3 choice (1
rd
Rafael: Theo: 8 Rafael: 7 Theo: 5
point) 10

Theo: 30 + 8 + 14 + 5 = 57 points

Nicole: 20 + 24 + 21 + 10 = 75 points

Rafael: 10 + 16 + 7 + 15 = 48 points

Nicole is the winner.

150
Example 3:

Contestant Rankings

A 5 4 1 1 3

B 4 1 5 2 2

C 3 5 4 3 1

D 2 2 2 4 5

E 1 3 3 5 4

Number of Voters 120 90 56 123 31

How to solve this:

Contestant Rankings

A 5 (1 4 (2 1 (5 1 (5 3 (3
point) points) points) points) points)

B 4 (2 1 (5 5 (1 2 (4 2 (4
points) points) point) points) points)

C 3 (3 5 (1 4 (2 3 (3 1 (5
points) point) points) points) points)

D 2 (4 2 (4 2 (4 4 (2 5 (1
points) points) points) points) point)

E 1 (5 3 (3 3 (3 5 (1 4 (2
points) points) points) point) points)

151
Number of 120 90 56 123 31
Voters

Contestant A: 120(1) + 90(2) + 56(5) + 123(5) + 31(3) = 1288 points

Contestant B: 120(2) + 90(5) + 56(1) + 123(4) + 31(4) = 1362 points

Contestant C: 120(3) + 90(1) + 56(2) + 123(3) + 31(5) = 1086 points

Contestant D: 120(4) + 90(4) + 56(4) + 123(2) + 31(1) = 1341 points

Contestant E: 120(5) + 90(3) + 56(3) + 123(1) + 31(2) = 953 points

Thus, using the Borda Count method of voting, contestant B wins.

3. Plurality by Elimination

∙ The plurality with elimination voting method is also known as an


instant run-off voting and sequential run-off voting.
∙ It is a preferential voting method and candidates that have the
least first place votes get eliminated until one candidate has
majority of first place votes.

152
Example 1:Let’s say we have a town of 20,000 people electing a mayor
using the Plurality with Elimination Voting Method. There are 4
candidates, candidate A, candidate B, candidate C, and candidate D.
The results of the election are shown below in a preference schedule:

Number of Voters 4000 7000 3000 6000

1st D C A B

2nd A B B D

3rd B D C C

4th C A D A

Because no candidate received a majority of first place votes,


the candidate with fewest first place votes is eliminated, which is
candidate C. The adjusted preference schedule is shown below:

Number of Voters 4000 7000 3000 6000

1st D B A B

2nd A D B D

153
3rd B A D A

Again, because no candidate has the majority of first place


votes, the candidate with the fewest first place vote is eliminated,
which is candidate D. The adjusted preference schedule is shown
below:
Number of Voters 4000 7000 3000 6000

1st A B A B

2nd B A B A

Finally, there is candidate who receives the majority of first place


votes, which is candidate B with 13,000 votes. So candidate B wins the
election using the Plurality with Elimination Voting Method.

Preference Ranking: A preference ranking is a voter’s order of preference of the


candidates.

Example 2: Three candidates are running for chairman of a institutional


organization. The preference ranking of the voters are as follows.

Number of Voters

16 5 10 8 10 7

Perona 1 1 2 3 2 3

Reyes 2 3 1 1 3 2

Santos 3 2 3 2 1 1

a)Who would win in a plurality election with a runoff between the top 2
finishers?
b)In a plurality election, could the seven voters who ranked Santos first and
Reyes second achieve a preferable outcome by voting strategically?

154
c) In a plurality election with a runoff between the top candidates, could the
seven voters who ranked Santos first and Reyes second achieve a preferable
outcome by voting strategically?

How to solve this:


a)We saw in the plurality election, the top two candidates are Perona and
Reyes. We have to see how Santos’ voters will change their votes. According
to the preference ranking, 10 will switch to Perona so Perona would have his
21 original votes and 10 more, giving him 31. Reyes would have his original
18 and another 7, giving him 25 votes. Perona would thus still win.
b)Yes, if the 7 voters had voters had voted for Reyes in the plurality election
instead of Santos, Reyes would have 25 votes, and so Reyes would have won.
These 7 votes prefer Reyes to Perona, and so this is a preferable outcome.
c)First, if the 7 voters switch their votes in the first round of voting, they can’t
change
who ends up in the runoff, since Reyes is already in the runoff. Second, in the
runoff, they prefer Reyes but voting him doesn’t affect the outcome, since
Perona still wins. Thus, the answer is no.

4. Pairwise Comparison Method


∙ Compare each two candidates head-to-head.
∙ Award each candidate one point for each head-to-head victory.
∙ The candidate with the most points wins.

Example 1:

Number of Voters 18 11 9 5 2

1st choice A C D B C

2nd choice B D C D B

3rd choice C B B C D

4th choice D A A A A

∙ Compare A to B.
155
⮚ 18 voters prefer A.
⮚ 11 + 9 + 5 + 2 = 27 voters prefer to B.
⮚ B wins the pairwise comparison and gets 1
point.
∙ Compare A to C.
⮚ 18 voters prefer A.
⮚ 11 + 9 + 5 + 2 = 27 voters prefer to C.
⮚ C wins the pairwise comparison and gets 1
point.

∙ Compare A to D.
⮚ 18 voters prefer A.
⮚ 11 + 9 + 5 + 2 = 27 voters prefer to D.
⮚ D wins the pairwise comparison and gets 1
point.
∙ Compare B to C.
⮚ 18 + 5 = 23 voters prefer B.
⮚ 11 + 9 + 2 = 22 voters prefer to C.
⮚ C wins the pairwise comparison and gets 1
point.

∙ Compare B to D.
⮚ 18 + 5 + 2 = 25 voters prefer B.
⮚ 11 + 9 = 20 voters prefer to D.
⮚ B wins the pairwise comparison and gets 1
point.

∙ Compare C to D.
⮚ 18 + 11 + 3 = 32 voters prefer C.
⮚ 9 + 5 = 14 voters prefer D.
⮚ C wins the pairwise comparison and gets 1
point.

Contestant Total Number of


Points
A 0

156
B 2

C 3

D 1

Therefore, C wins using pairwise comparison method.

Example 2:Let’s say we have a town of 20,000 people electing a mayor using
the Plurality with Elimination Voting Method. There are 4 candidates,
candidate A, candidate B, candidate C, and candidate D.
Number of Voters 4000 7500 3000 5500

1st D C A B

2nd A B B D

3rd B D C C

4th C A D A

∙ Compare A to B.
⮚ 4000 + 3000 = 7000 voters prefer A.
⮚ 7500 + 5500 = 13000 voters prefer to B.
⮚ B wins the pairwise comparison and gets 1
point.

∙ Compare A to C.
⮚ 4000 + 3000 = 7000 voters prefer A.
⮚ 7500 + 5500 = 13000 voters prefer to C.
⮚ C wins the pairwise comparison and gets 1
point.

∙ Compare A to D.
⮚ 3000 voters prefer A.
⮚ 4000 + 7500 + 5500 = 17000 voters
prefer to D.
⮚ D wins the pairwise comparison and gets 1
point.

157
∙ Compare B to C.
⮚ 4000 + 3000 + 5500 = 12500 voters
prefer B.
⮚ 7500 voters prefer to C.
⮚ B wins the pairwise comparison and gets 1
point.
∙ Compare B to D.
⮚ 7500 + 3000 + 5500 = 16000 voters
prefer B.
⮚ 4000 voters prefer to D.
⮚ B wins the pairwise comparison and gets 1
point.

∙ Compare C to D.
⮚ 7500 + 3000 = 10500 voters prefer C.
⮚ 4000 + 5500 = 9500 voters prefer D.
⮚ C wins the pairwise comparison and gets 1
point.

Contestant Total Number of


Points
A 0

B 3

C 2

D 1

Therefore, B wins using pairwise comparison method.

158
159
160
161
162
CHAPTER MATHEMATICS AS A TOOL :
GRAPH THEORY
7
LEARNING OUTCOMES:

At the end of the chapter, the students are expected to :


∙ Define Graph theory, nodes, edges, paths and different kind
of paths.
∙ Determine the Euler and Hamiltonian paths and circuits.
∙ Solve real life problems using concepts, algorithms and
theorems of Graph Theory

LESSON 1 : Basic Terms

Graph

∙ A diagram that contains information and depicts connection and


relationship between the various parts of the diagram.

Examples:

163
1.) Road Map
2.) Circuit Diagram
3.) Flow Chart
4.) Transportation Route
5.) Tree Diagram

Essential Features of a Graph

1.) The Objects – referred to as the nodes or vertices


2.) Edges – the connecting lines

Example. Consider the following graphs:

1.)
The graph has 4 vertices and 4 edges

2.)

The graph has 4 vertices and 3 edges.

3.)

The graph has 5 vertices and 8 edges

164
Note: From example 3;

1. Edge e4 is known as Loop.


Loop is an edge that connects a vertex to its self.
2. Edges e7 and e8 are called multiple edges or parallel
edges. Parallel Edges – are edges that connect the same
vertices.

Path
- A path in graph theory is a sequence of edges.

Example: Consider the following graphs below.

G1:

Paths Length
1.) ade 3
2.) adc 3
3.) bce 3
4.) abced 5
5.) adecb 5

Note: 1. A path can repeat edges.


2. The length of a path refers to the number of
edges connected.

165
3. If the direction is not indicated in the graph by an
arrow, the movement is can be in any direction to
find a path (you can move backward and forward).
However if there is an arrow indicating direction the
movement in finding a path is in accordance with the
indicated arrow .

G2
:

Paths Length
1. e4 e3 2
2. e3 e2 e4 3
3. e1 e5 e4 3
4. e1 e5 e4 e3 e2 5
5. e1 e5 e5 e4 4

Vertex Sequence of a Path


- a path is written in terms of edges. A path determines a
sequence of vertices.

G3:

Path Length Vertex Seq. # of Edges # of Vertex

166
1. adeecbd 7 XYZZZWYZ 7 8
)
2. cbdec 5 ZWYZZW 5 6
)
3. eecbdecc 8 ZZZWYZZWZ 8 9
)

Note: 1.) The number of vertices in vertex sequence is always one


larger than the number of edges in the path.
2. ) If a path passes through a loop, the vertex of the loop is
repeated in the vertex sequence.

Brief History of Graph Theory

-From TeroHarju Lecture Note

Graph theory may be said to have its beginning in 1736 when EULER considered
the (general case of the) Königsberg bridge problem: Does there exist a walk
crossing each of the seven bridges of Königsberg exactly once?(Solution
Problema
tisadgeometriamsituspertinentis,CommentariiAcademiaeScientiarumImpe
rialisPetropolitanae 8 (1736), pp.128-140.) It took 200 years before the first book
on graph theory was written. This was
“TheoriederendlichenundunendlichenGraphen” (Teubner,Leipzig,1936) by
KÖNIG in 1936. Since then graph theory has developed into an extensive and
popular branch of mathematics, which has been applied to many problems in
mathematics, computer science, and other scientific and not- so- scientific areas.
For the history of early graph theory, see N.L. BIGGS,
R.J. LLOYD AND R.J. WILSON, “Graph Theory1736 – 1936”, Clarendon Press,
1986.-

167
The Konigsberg Bridge Problem

Closed Path

- A closed path is said to be closed path if the first and the last vertices
of its vertex sequence are the same.

Example:

G4:

Path Vertex Sequence


1.) acef LMNOP (not closed)
2.) acb LMNL (closed)
3.) fed PONO (not closed)
4.) ecabd ONMLNO (closed)

Cycle
- a path is called a cycle if the following conditions are
satisfied: a.) the path is closed,
b.) the path repeat no edges,
c.) the vertices of the vertex sequence of the path are all distinct except
for the 1st and last vertices which are the same vertices.

168
Example:

G5:

Path Vertex Sequence


a.) e1 e2 e5 e3 ABDCB (not closed, not a
cycle)
b.) e4 e5 e2 e3 CCDBC (closed, not a
cycle)
c.) e2 e3 e5 DBCD (closed, cycle)
Connected Graph

- a graph is called connected graph if for any two given vertices there is
a path connecting them.

Example:

G6: Consider the following graphs;

not connected

connected

169
connected

Complete Graph

-is a connected graph where every pair of vertices is joined by an


edge.
Example:
G7:

a.) b.) c.)

Simple Graph

-a graph is simple if it has neither loop nor parallel edges.

Example:
G8:

a.) Simple graph

Not a simple graph


b.)

170
c.) Not a simple graph

Degree of a Vertex

- The degree of a vertex is defined as the number of


edges connected to the vertex.
- If a graph contains a loop, the loop contributes 2 to the
degree of the vertex.

Example:

G9: Vertex Degree

A 3
B 6
a.) C 4
D 5

Acyclic Graph

- A graph is called acyclic graph if it has no cycle.

Example:

G10:
a.) No cycle; acyclic

171
b.)

Cycle: CDHG - wsrv

Not Acyclic

Weighted Graph

- A graph whose edges are assigned with weights. Weight may represent
mileage, time, cost, or some other quantities.

Example: Consider the transportation route of a salesman;

G11:

Note: The vertices correspond to the different cities while edges represent the
distances in kilometers.

Tree
- An acyclic connected graph.
- Properties:
a.) acyclic graph
b.) no cycle path
c.) connected graph
-Forest -refers to several trees

172
Example:
G12:
Acyclic, connected
-tree

Spanning Tree

-a sub-graph tree of a graph that contains all the vertices of the


graph.

Example:

G13:

G13.1 G13.2

G13.1 and G13.2 are spanning trees of G13.

Note: If a graph has an n-edges then there are n! Spanning tree in the graph.

173
Example:

G13.3 5 edges
5! = 120 spanning tree

Minimal Spanning Tree

- the minimal spanning tree of a graph is a spanning tree of the graph with a
minimum total weights
- a connected graph has always at least one minimal spanning tree.

Example:

G14:

G14.1

G14.1 is the minimal spanning tree of G14,i.e., 5+8+10 (23 total


weights)

174
Two Special Circuits

A. Euler Circuit:
- A closed path in a graph which uses each of the edges exactly
once,
- Named after Leonard Euler(April 15, 1707 –September
18, 1783), a Swiss Mathematician and Physicist. He
started working on graphs from year 1736.

Euler Theorem:
-The graph has an Euler Circuit if the graph is connected and
the degree of the vertices must be even.

Euler Path
- a path that uses each edge of a graph exactly once.

Euler Path Theorem


-a connected graph has an Euler path if and only if the graph
has two vertices of odd degree with all other vertices of even degree.
Example:
G15:
a.) The graph is connected,
Contains Euler path,
Two of the vertices have odd degree,
By Euler Theorem, the graph has no Euler Circuit

b.) Not connected,


Does not contain Euler path and
Euler circuit.

175
c.)
Connected, contains Euler path,
All vertices has an even degree,
Contains Euler Circuit.

Solution in Konigsberg Bridge Problem

Konigsberg bridge problem:


Does there exist a walk crossing each of the seven bridges of
Königsberg exactly once?

Graph Representation:
G16:

By Euler Theorem, the graph contains vertices having an odd degree; the
graph has no Euler Circuit. There is no closed path that will bring us to each
island by crossing each bridge exactly once.

Therefore, the Konigsberg Bridge problem has no solution.

176
Fleury’s Algorithm
-Is used to find an Euler Circuit in a graph if the graph has one.

Steps:
1. Select any of the vertices in the graph as the starting point.
2. Select any edge connected to the vertex selected in step 1. Remove the
edge. (The removal of the edge must not disconnect the starting vertex
or the starting point). After the removal of the edge a new vertex is
reach.
3. Select an edge connected to the new vertex and repeat step 2.
4. Repeat step 3 until the starting vertex is reach.

Example:
G17:

Let Vertex X be the starting point.


Remove edge f

Remove edge d

Remove edge e

Remove edge c

177
Remove edge b

Remove edge a

Therefore, the Euler Circuit is


- fdecba (with a vertex sequence: XYZZWYX).
B. Hamilton Circuit
- A closed path which uses each vertex of the graph exactly
once, except for the last vertex which duplicate the 1st vertex.
- Also known as Hamiltonian Circuit.
- Named after Sir William Rowan Hamilton (1805-1865)
He marketed a puzzle in the mid 1800 in the form of a
dodecahedron that contains the name of a city in each
corner. The problem is to visit each city exactly once by
travelling along the edges and be able to return to the
starting city. (D.S. Malik and M.K. Sen)

The Puzzle

Example:
G18:
178
The graph has Hamilton circuit:
V1V2V3V4V5
a.)

b.) The graph has no Hamilton circuit,

since it will make use of the middle


vertex (V) twice.

Dirac’s Theorem
- Consider a connected graph with at least 3 vertices and no multiple
edges. Let n be the number of vertices in the graph. If every vertex has
degree of at least n/2, then the graph must be HAMILTONIAN.

Example:
G19:

Algorithm used to find a Hamiltonian Circuit

Greedy Algorithm
- Also known as shortest path algorithm,

179
- Developed by Dijkstra.
Steps:
1. Choose a vertex to start at, then travel along the connected edge that has the
smallest weight.( if two or more vertices have the same weight, pick any one)
2. After arriving at the next vertex, travel along the edge of smallest weight
that connects to a vertex not yet visited. Continue this process until you have
visited all vertices.
3. Return to the starting vertex.

Example: Consider Graph 11.

G11:

Let C1 be the starting point, choose edge 80,

From C2, choose edge 90,

180
From C4, choose edge 160 (in this case edge 80 is not advisable since we
will cross C3 twice),

From C7, choose edge 110 instead of edge 70 (since we will cross C5 twice if we
choose edge 70),

Frrom C8, choose edge 105,

181
From C5 choose edge 60,

From C6, choose edge 200

From C3, choose edge 90,

182
The Hamilton Circuit is now completed, with a
vertex sequence: C1C2C4C7C8C5C6C3C1
total weight : 80+90+160+110+105+60+200+90=835.
The Edge-picking Algorithm
Steps:
1. Mark the edge of the smallest weight in the graph.(if two or more edges
have the same weight, pick only one)
2. Mark the edge of next smallest weight in the graph, as long as it does
not
complete a circuit does not add a third marked edge to a single vertex.
3. Continue this process until you can no longer mark any edges. Then
mark the final edge that completes the Hamiltonian circuit.
Planar Graph
-is a graph that can be drawn so that no edge intersects each
other (except at vertices)

Example:
G20: G20.1

a.)

G20 is a planar graph since it can be re-drawn (G20.1) without any


intersecting edges.
b.)
G20.2 G20.3

182
G20.2 is NOT a planar graph since it cannot be re-drawn
(G20.3) without any intersecting edges.

Matrix Representation of a Graph

Rule: If a graph has N vertices its matrix representation has NxN shape,
denoted by M.

Example: Write the matrix representation of the graph below.


G21:

W X Y Z
W 0 2 1 1
M= X 2 0 0 0
Y 1 0 0 1
Z 1 0 1 2

Note: 1.) The entries indicate the edges between the vertices.
M(W,X) = 2 means that there are 2 edges that connects W &
X.
M(X,Y) = 0 means that there is no edge that connects X & Y.
2.The diagonal indicates the loop/s in the graphs.
M(Z,Z) =2 means that there is a loop that is connected to vertex Z
3.)The degree of the vertex is equal to the sum of the entries in its row
or column.
Deg(W) = 4 (sum of its column entry or of its row
entries)

184
4.) The matrix is symmetric with respect to its
diagonal.

Adjacency Matrix of Simple Graph


Theorem: If M is the adjacency of a simple graph, then the row*
column entry of Mn is equal to the number of length n from vertex from row to
the vertex in column, n=1,2,3…

Example. Consider the Graph


G22:

How many paths of length 2 can be used to reach a vertex from


another?

Solution: Path of Length 2 = M2 = MxM


The Matrix (M) of the is;
W X Y Z
W 0 1 1 1
M
X 1 0 1 1
= Y 1 1 0 1
Z 1 1 1 0

185
Determine M 2 = M x M

W X Y Z W X Y Z
W 0 1 1 1 W 0 1 1 1
MxM= X 1 0 1 1 x X 1 0 1 1
Y 1 1 0 1 Y 1 1 0 1
Z 1 1 1 0 Z 1 1 1 0

W X Y Z
W 3 2 2 2
M 2= X 2 3 2 2
Y 2 2 3 2
Z 2 2 2 3

M2(W,W)= 3, means there 3 paths of length 2 that connects vertex W to


itself. That is; e1e1, e6e6, e4e4.
M (X,Z)=2 means there are 2 paths of length 2 that connects vertex x
2

to vertex Z, that is; e1e4 and e2e3.


Since the Matrix is symmetric, we can consider the upper
diagonal
or the lower diagonal. Therefore the total number of paths of length
is 3+3+3+3+2+2+2+2+2+2 =24 paths.

Note: Paths of length 3 is M3 of length 4 is M4.

Lesson 2: Applications of Graph Teory

The Pipeline Problem


The MAYNILAD is considering 8 cities to be connected by a pipeline. The
distances (in km) between cities are given in the graph below:

186
Determine the minimum length of pipe that MAYNILAD is needed to connect
the 8 cities. Use the minimal spanning tree.

Solution:

Total weight of the minimal spanning tree


= 80 + 90+ 80+150 +60+70+105 = 635

Therefore, Maynilad needs 635km length of pipeline to connect the 8 cities.

The Map Coloring Problem

The Four Color Theorem


- Every map in the plane consisting of connected region without a hole
can be colored with four different colors without coloring two adjacent
regions with the same color.
Example: Consider the Map that shows some of the states of US.

187
How the map should be colored with 4 colors if no two adjacent states
should have with the same color?

Solution:
Draw the graph representation of the
map. a.) Vertices are the regions or
states
b.) Edges –to connect two vertices if the states or regions corresponding to
these vertices are adjacent.
Graph representation,

Use 4 colors to be assigned to its vertices.


Assigned 2 different colors to any 2 adjacent vertices.

188
Travelling Salesman Problem (Hamiltonian Problem)

Example: In 1859, Sir William Rowan Hamilton marketed a game called Around the
World. The game consisted of a regular dodecahedron made of wood. Each corner bore
the name of a famous city of the world. The game was to find a path starting at any city,
travelling along the edges of the dodecahedron, visiting each city exactly once and
returning to the starting city. The diagram below represents the game. (Source: D.S.
Malik and M.K. Sen)

Around The World


Solution:
Using the Hamiltonian Circuit:

189
Therefore, one of the Hamiltonian Circuit that can be an answer to
the puzzle is the path with vertex sequence of

V1V2V3V4V5V14V13V12V11V10V9V8V7V17V18V19V20V16V15V6V1.

190
191
192
193
194
195
196
References:

Altares, Priscilla S., Copo, A. R., et al, (2013)Elementary Statistics with


Computer Applications, REX Book Store
Baltazar, E. C., Ragasa, Carmelita, Evangelista, J. (2018).
MATHEMATICS in the Modern World, C& E Publishing, Inc.
F.J. MacWilliams, and N.J.A. SLoane, (1981), "The Theory of Error-
Correcting Codes", Bell Laboratories, Murray Hill,

J.A. Buchmann,(2002), "Introduction to Cryptography", Springer-


Verlag, New York,

Lipmann, David, (2007), “Math in Society, Edition 2.5”, Piece College


Ft Steilacom
Development.

Malik and Sen, (2010), “ Discrete Mathematics”, C& E Publishing, Inc.

Nocon, R. & Nocon E. (2018), “Essential Mathematics for the Modern


World” , C& E
Publishing, Inc.

Punsalan, Twila G., Uriarte, Gabriel G. (2013) STATISTICS: A


Simplified Approach, REX Book Store.

Parreno, Elizabeth B., Jimenez, Ronel O., ( 2006) BASIC


STATISTICS: A Worktext, C &E Publishing, Inc.
W.C. Huffmann, and V. (2003) "Fundamentals of Error Correcting
Pless, Cambridge Codes",
University Press,

Website :
www. Wikipedia

197
19
8
GLOSSARY
A
Acyclic graph - if graph has no cycle.
Apportionment - Is a method of dividing a whole into various parts.
Apportionment principle – when a new representative is added to a sub –
group
due to an increase in population.
Argument - reasons offered or against something.

B
Borda Count Method - Award points to candidates based on preference
Schedule, then declare the winner to the candidate with
most points.
C

Ciphertext - refers to the coded message.


Closed path - if the first and the last vertices of its vertex sequence
are the same.
Communication channel - is the physical medium through which
information is
transmitted.

Communication system - is usually composed of sender (or message


source),
communication channel, and the receiver.
Compound proposition - conveys two or more
idea. Conclusion – last step in the reasoning
process.
Conjecture - is an educated guess based on repeated observation of a
particular Process or pattern.
Connected graph - if for any two given vertices there is a path connecting
Connected graph - if for any two given vertices there is a path
connecting them.
Continuous variables - can assume an infinity of many possible values
Corresponding to the point on a line interval.
Correlation - measures the association or the strength of the
relationship between two variables say x and y.
Counterexample - a statement that disproves the conjecture.
Cryptography - Is a method of making and breaking of secret
codes.
199
cycle - if the path is closed and the path repeat no edges.

D
Data - a result of experiment, observation, investigation and often
appears as numerical figure.
Decile- is a measure of position that divides the ordered observations or
score into ten equal parts.
Decoding - is the opposite process of encoding.
Decryption - is the process of returning/converting back the coded
message Into plain text.
Deductive reasoning - process of solving problems by applying
premises , syllogism and conclusion.
Degree of a vertex - is defined as the number of edges connected to the vertex.
Descriptive statistic - division of statistics that summarizes or describes the
important characteristics of a given data set.
Discrete variables - can assume a finite or countable number of values.
Dispersion - measure the degree of clustering of data about a central
point.

E
Edges – the connecting lines
Encoding - is transforming messages into bits of message that is suitable
to the communication channel.
Encryption - Is the process of transforming plain text into code form
using a certain algorithm.
Euler Circuit: - a closed path in a graph which uses each of the edges exactly
once,
Existential quantifier - means “ there exists “

F
Fibonacci sequence - is a series of numbers where a number us found by
adding the two numbers before it.

G
Golden ratio - also called the golden section, golden mean or divine
proportion.
Graph - a diagram that contains information and depicts connection
and relationship between the various parts of the diagram.
Graphene – means writing.

200
H

Hamilton Circuit - a closed path which uses each vertex of


the graph exactly once, except for the last
vertex which duplicate the 1st vertex.
Hamilton plan method - given the number of seat in the US
congress will be
apportioned between states proportionally to
their population.
Huntington – Hill apportioned method - the method that make use
of equal
proportion.
I
Inductive reasoning - same method would work for any similar type of
problem to arrived at a conclusion.
Inferential statistics - aims to give information about the population by
studying the characteristics of the sample drawn
from it.
Interval - it gives meaningful amount of differences between data but
does not
have a true – zero starting point.

J
Jefferson plan method - is a method that uses a modified standard
divisor the
arrives at the correct or exact
numbers of representative using
trial and error .

Key - refers to the strings of information that is used to reveal the


encrypted message into readable form.
Krypto - means hidden.

201
L

Language - Is the fundamental tool that bridges the gap


among people from varying origins and
culture.
Linear Regression - is very powerful tool in predicting the value
of one
variable to the given value of another
variable .
Loop - is an edge that connects a vertex to its self
M
Mean - sum of all the values in the observation divided by the total
number of observation.
Median -is considered as the physical middle point of the distribution.
Minimal spanning tree - is a spanning tree of the graph with a minimum
total weights.
Mode - is the most occurring value in the distribution.
Modular Arithmetic - is an operator (mod), which seeks for a remainder
when two numbers are divided.

Nominal data - data that consist names, labels or


categories only. Normal curve - is a symmetrical curve
having a bell-like shape.
O
Objects – referred to as the nodes or vertices
Ordinal data - measurements which deals order or rank but
difference between rank does not exist.

P
Pairwise Comparison Method - compare each two candidates head-to-
head and award each candidate one point for each head-
to head victory.
Parallel Edges – are edges that connect the same vertices.
Parameter - numerical measures that describe the population of
interest.
Parity check - encoding scheme where in a single bit is added to the
message as

202
redundancy bit.
Path - in graph theory is a sequence of edges.
Pearson Product- Moment Correlation - Is the most familiar sort of
statistical tool in quantifying the linear relationship
between two random variables, x and y.
Percentiles – is a measure of position that divides the ordered
observations or score distribution into 100 equal parts.
Plain text - refers to the original message.
Plurality by Elimination - a voting method which is also known as an
instant
run-off voting and sequential run-off voting
Plurality Method - the winner is the candidate or choice with the
most votes.
Polya’s strategy - a four step problem solving strategy.
Population - set of complete collection of all possible values of the
variable. Premises – assertions that serve as the basis for an
argument.
Proposition - is a declarative statements that may expressed an idea
which can be true or false but not both.

Q
Quantiles - is a measure of position determines the position of a single
value in relation to other values in a sample or a
population data set.
Quartile- is a measure of position that divides the ordered observations
or score distribution into four equal parts.

Range - the highest value minus the lowest value.


Ratio - a modified interval level to include the starting point zero.
S
Sample - portion of the population.
Set - is a collection of, numbers, people , letters of the
alphabet. Shift cipher - uses shift in forming the key of
cryptography.
Simple graph - a graph is simple if it has neither loop nor parallel edges.
Simple proposition - means single idea.
Source coding - is defined as converting the messages from the
sender into bits suitable to the communication
channel.
Spanning tree - a sub-graph tree of a graph that contains all the vertices of the
202
graph.

Standard divisor - the number of voters represented by each


representative.
Standard quota - the whole part of the quotient when the population
of the sub – group is divided by the standard
divisor.
Statistics – is a collection, presentation ,analysis and interpretation of
data .
Strategic Voting - when a person votes in a way that does not reflect
his or her true preferences in an attempt to improve
the outcome of the election.
Syllogism - argument composed of two statements followed by a
conclusion. Synonyms - different words with same meaning.

T
Tree - an acyclic connected graph.

U
Universal quantifier - means “ for all “ or “ for every “

V
Variable - attribute of interest observable of each entity in the
universe. Variance - is the square of the deviation of data sets
from its mean.
Vertex Sequence - a path is written in terms of edges.
W

Weighted graph - a graph whose edges are assigned with weights. Weight
may represent mileage, time, cost, or some other quantities.

Z-score - these standard scores tell how far a raw score is from the
mean in standard deviation units.

204
20
5

You might also like